28.06.2020

Veden ja elektrolyyttien vaihto on normaalia. Vesi-suolan aineenvaihdunta ihmiskehossa: miten se toimii. Elektrolyyttitasapainon hoito


Vesi-suola-aineenvaihdunta on veden ja elektrolyyttien kehoon pääsyn, niiden jakautumisen sisäisessä ympäristössä ja kehosta erittymisen prosessien kokonaisuus.

Vesi-suolan aineenvaihdunta ihmiskehossa

Vesi-suola-aineenvaihduntaa kutsutaan joukko prosesseja, joissa vettä ja elektrolyyttejä tulee kehoon, niiden jakautuminen sisäiseen ympäristöön ja erittyminen kehosta.

Terve ihminen säilyttää tasa-arvoisena elimistöstä vapautuvan ja siihen saapuvan veden määrät vuorokaudessa, mikä on ns. vesitasapainoa kehon. Voit myös harkita elektrolyyttitasapainoa - natriumia, kaliumia, kalsiumia jne. Terveen ihmisen keskimääräinen vesitase levossa on esitetty taulukossa. 12.1 ja elektrolyyttitasapaino taulukossa. 12.2.

Ihmiskehon vesitasapainoparametrien keskiarvot

Taulukko 12.1. Ihmiskehon vesitasapainoparametrien keskiarvot (ml/vrk)

Veden kulutus ja tuotanto

Veden vapautuminen

Juominen ja nestemäinen ruoka

1200

Virtsan kanssa

1500

Kiinteää ruokaa

1100

Sen kanssa

500

Endogeeninen "hapetusvesi"

300

Uloshengitetyn ilman kanssa

400

Ulosteiden kanssa

100

Kuitti yhteensä

2500

Kokonaismäärä

2500

Ruoansulatuskanavan nesteiden sisäinen kierto (ml/vrk)

Eritys

Reabsorptio

Sylki

1500

Mahalaukun mehu

2500

Sappi

500

Haima mehu

700

Suoliston mehu

3000

Kaikki yhteensä

8200

8100

Yhteensä 8200 - 8100 = vettä ulosteessa 100 ml

Tiettyjen aineiden keskimääräinen päivittäinen metabolinen tasapaino ihmisellä

Taulukko 12.2 Tiettyjen aineiden keskimääräinen päivittäinen metabolinen tasapaino ihmisellä

Aineet

Sisäänpääsy

Valinta

ruokaa

aineenvaihduntaa

virtsa

ulosteet

hikeä ja ilmaa

Natrium (mmol)

155

150

2,5

2,5

kalium (mmol)

5,0

Kloridi (mmol)

155

150

2,5

2,5

Typpi (g)

Hapot (meq)

haihtumaton

haihtuvia

14000

14000

Erilaisten häiritsevien vaikutusten alaisena(muutokset ympäristön lämpötilassa, erilaiset fyysisen toiminnan tasot, muutokset ravitsemusmallissa) yksittäiset saldoindikaattorit voivat muuttua, mutta itse saldo pysyy ennallaan.

Patologisissa olosuhteissa esiintyy epätasapainoa, jossa vallitsee joko veden kertyminen tai menetys.

Kehon vesi

Vesi on kehon tärkein epäorgaaninen komponentti, joka varmistaa yhteyden ulko- ja sisäympäristön välillä sekä aineiden kuljetuksen solujen ja elinten välillä. Orgaanisten ja epäorgaaniset aineet, vesi on tärkein ympäristö aineenvaihduntaprosessien kehittymiselle. Se on osa erilaisia ​​järjestelmiä orgaaniset aineet.

Yksi gramma glykogeenia sisältää esimerkiksi 1,5 ml vettä, jokainen gramma proteiinia sisältää 3 ml vettä.

Sen osallistumisella muodostuu sellaisia ​​rakenteita kuin solukalvot, veren kuljetuspartikkelit, makromolekyyliset ja supramolekyyliset muodostelmat.

Aineenvaihdunnan ja vedyn hapettumisen prosessissa muodostuu substraatista erotettuna endogeeninen "hapetusvesi", Lisäksi sen määrä riippuu hajoavien substraattien tyypistä ja aineenvaihdunnan tasosta.

Joten levossa hapettumisen aikana:

  • 100 g rasvaa tuottaa enemmän kuin 100 ml vettä,
  • 100 g proteiinia - noin 40 ml vettä,
  • 100 g hiilihydraatteja - 55 ml vettä.

Lisääntynyt katabolia ja energia-aineenvaihdunta johtaa jyrkkä nousu muodostunut endogeeninen vesi.

Ihmisen endogeeninen vesi ei kuitenkaan riitä tarjoamaan vesipitoista ympäristöä aineenvaihduntaprosesseille, erityisesti aineenvaihduntatuotteiden erittymiselle liuenneessa muodossa.

Erityisesti proteiinin kulutuksen lisääntyminen ja vastaavasti niiden lopullinen muuttuminen ureaksi, joka poistuu elimistöstä virtsan mukana, johtaa ehdottomaan tarpeeseen lisätä munuaisten nestehukkaa, mikä edellyttää lisääntynyttä saantia kehon.

Syömällä pääosin hiilihydraatteja, rasvaisia ​​ruokia ja pienellä määrällä NaCl:a elimistössä, elimistön vedentarve on pienempi.

    Terveen aikuisen päivittäinen vedentarve on 1-3 litraa.

    Veden kokonaismäärä ihmisen kehossa vaihtelee välillä 44-70 % kehon painosta tai noin 38-42 litraa.

    Sen pitoisuus eri kudoksissa vaihtelee 10 %:sta rasvakudoksessa 83-90 %:iin munuaisissa ja veressä; iän myötä kehon veden määrä vähenee sekä liikalihavuuden myötä.

    Naisilla on pienempi vesipitoisuus kuin miehillä.

Kehon vesi muodostaa kaksi vesitilaa:

1. Solunsisäinen (2/3 vedestä).

2. Solunulkoinen (1/3 kokonaisvedestä).

3. Patologisissa olosuhteissa ilmaantuu kolmas vesitila - ruumiinontelovesi: vatsa, keuhkopussin jne.

Solunulkoinen vesitila sisältää kaksi sektoria:

1. Suonensisäinen vesisektori, ts. veriplasmaa, jonka tilavuus on noin 4-5 % kehon painosta.

2. Interstitiaalinen vesisektori, joka sisältää 1/4 kaikesta kehon vedestä (15 % kehon painosta) ja on liikkuvin, joka muuttaa tilavuutta kehon ylimäärän tai puutteen vuoksi.

Kaikki kehon vesi uusiutuu noin kuukaudessa ja solunulkoinen vesitila viikossa.

Kehon ylihydraatio

Liiallinen veden saanti ja muodostuminen riittämättömän vähäisellä erityksellä kehosta johtaa veden kertymiseen ja tämä vesitasapainon muutos on ns. ylihydraatio.

Ylihydratoituessa vesi kerääntyy pääasiassa interstitiaaliseen vesisektoriin.

Vesimyrkytys

Ilmenee huomattava ylihydraatioaste vesimyrkytys .

Samalla interstitiaalisessa vesisektorissa osmoottinen paine tulee alhaisemmaksi kuin solujen sisällä, ne imevät vettä, turpoavat ja myös osmoottinen paine niissä laskee.

Lisääntyneen herkkyyden seurauksena hermosolut osmolaarisuuden vähenemiseen, vesimyrkytys voi liittyä levottomuuteen hermokeskukset ja lihaskrampit.

Kehon kuivuminen

Riittämätön veden saanti ja muodostuminen tai sen liiallinen vapautuminen johtaa vesitilojen pienenemiseen, pääosin, välisektori, jota kutsutaan kuivuminen.

Tähän liittyy veren paksuuntuminen, sen reologisten ominaisuuksien heikkeneminen ja heikentynyt hemodynamiikka.

Veden puute kehossa 20 % painosta johtaa kuolemaan.

Kehon vesitasapainon säätely

Vesitasapainon säätöjärjestelmä tarjoaa kaksi pääasiallista homeostaattista prosessia:

    Ensinnäkin nesteen kokonaistilavuuden ylläpitäminen kehossa ja

    toiseksi veden optimaalinen jakautuminen vesitilojen ja kehon sektoreiden välillä.

Veden homeostaasia ylläpitäviä tekijöitä ovat mm nesteiden osmoottinen ja onkoottinen paine vesitiloissa, hydrostaattinen ja hydrodynaaminen verenpaine, histohemaattisten esteiden ja muiden kalvojen läpäisevyys, elektrolyyttien ja ei-elektrolyyttien aktiivinen kuljetus, munuaisten ja muiden erityselinten toimintaa säätelevät neuroendokriiniset mekanismit sekä kuten juominen ja jano.

Vesi-suola-aineenvaihdunta

Elimistön vesitasapaino liittyy läheisesti elektrolyyttiaineenvaihduntaan. Mineraalien ja muiden ionien kokonaispitoisuus luo tietyn määrän osmoottista painetta.

Yksittäisten mineraali-ionien pitoisuus määrää toimiva tila kiihottavat ja ei-ärsyytyvät kudokset sekä biologisten kalvojen läpäisevyyden tila - siksi on tapana sanoa O vesi-elektrolyytti(tai suolaliuosta)vaihto.

Vesi-elektrolyyttiaineenvaihdunta

Koska mineraali-ioneja ei syntetisoidu elimistössä, ne on saatava kehoon ruoan ja juoman kautta. Säilyttääksesi elektrolyyttitasapainon ja vastaavasti elintärkeän toiminnan, kehon pitäisi saada päivässä noin 130 mmol natriumia ja klooria, 75 mmol kaliumia, 26 mmol fosforia, 20 mmol kalsiumia ja muita alkuaineita.

Elektrolyyttien rooli kehon toiminnassa

Homeostaasiin elektrolyytit vaativat useiden prosessien vuorovaikutusta: pääsy elimistöön, uudelleenjakautuminen ja kerrostuminen soluihin ja niiden mikroympäristöön, erittyminen kehosta.

Pääsy kehoon riippuu koostumuksesta ja ominaisuuksista elintarvikkeita ja vesi, niiden imeytymisen ominaisuudet maha-suolikanavassa ja suolistoesteen tila. Kuitenkin huolimatta suurista vaihteluista määrässä ja koostumuksessa ravinteita ja vesi, vesi-suolatasapaino sisällä terveellinen keho pysyy tasaisesti yllä erittymisessä erittyvien elinten kautta tapahtuvien muutosten ansiosta. Munuaisilla on tärkeä rooli tässä homeostaattisessa säätelyssä.

Vesi-suola-aineenvaihdunnan säätely

Vesi-suola-aineenvaihdunnan säätely, kuten useimmat fysiologiset säätelyt, sisältää afferentit, keskus- ja efferentit linkit. Afferenttia linkkiä edustaa verisuonikerroksen, kudosten ja elinten reseptorilaitteiden massa, joka havaitsee osmoottisen paineen, nesteiden tilavuuden ja niiden ionisen koostumuksen muutokset.

Tämän seurauksena keskushermostoon luodaan yhtenäinen kuva kehon vesi-suolatasapainon tilasta. Keskeisen analyysin seurauksena on muutos juoma- ja syömiskäyttäytymisessä, ruuansulatuskanavan ja eritysjärjestelmän (ensisijaisesti munuaisten toiminnan) rakennemuutos, joka toteutetaan efferentin säätelylinkin kautta. Jälkimmäisiä edustavat hermostuneet ja suuremmassa määrin hormonaaliset vaikutukset. julkaistu

Penzan osavaltion yliopisto

lääketieteellinen koulu

teknisen ja sähkötekniikan osasto

kurssi "Extreme and sotilaslääketiede"

Vesi-elektrolyyttiaineenvaihdunta

Kokoonpano: lääketieteen kandidaatti, apulaisprofessori Melnikov V.L., vanhempi opettaja Matrosov M.G.

Vesi-elektrolyyttiaineenvaihdunta

1. Veden vaihto kehossa

Elimistön aineenvaihduntaprosessien normaalia kulkua varten sekä normaaleissa että patologisissa olosuhteissa tarvitaan oikea vesiympäristön kokonaistilavuus.

Veden kokonaistilavuus vastasyntyneellä on 80% ruumiinpainosta, aikuisella - 50-60%, vaihtelut riippuvat kehotyyppi, sukupuoli ja ikä. Tästä arvosta 40 % on peräisin solunsisäinen(solunsisäinen) ja 20 % solunulkoinen(solunulkoiset) tilavuudet.

Solunsisäinen neste on protoplasman orgaaninen osa. Verrattuna ekstrasellulaariseen sektoriin, solun sisällä havaitaan enemmän korkeatasoinen proteiinia ja kaliumia ja alhaisempia natriumpitoisuuksia. Tämä ero ionikonsentraatiossa syntyy kalium-natriumpumpun toiminnasta, joka tarjoaa hermo-lihasrakenteiden kiihtyvyyteen tarvittavan biosähköisen potentiaalin. Plasmasta soluun saatu vesi sisältyy kaikkiin biokemiallisiin prosesseihin ja vapautuu siitä vaihtuvan veden muodossa; Tämä koko sykli kestää 9-10 päivää. Vauvoilla tämä sykli kestää intensiivisempien redox-prosessien vuoksi 5 päivää.

Solunulkoinen tilavuusvesi jaettu kolmelle vesisektorille: intravaskulaarinen, interstitiaalinen ja transsellulaarinen.

1. Suonensisäinen sektori koostuu plasman tilavuudesta ja punasoluihin sitoutuneesta vedestä. Tavanomaisen punasoluihin äskettäin tulevan veden vaihdon lisäksi vaihtoveteen (katso edellä), osa punasoluista voi vapautua dehydraation aikana, ja ylihydraation aikana tapahtuu päinvastainen prosessi. Jos otamme huomioon, että erytrosyyttien massa on jopa 30 mg/kg ruumiinpainoa, niin punasoluihin sitoutuneen veden tilavuus on suunnilleen 2100 ml. Kun otetaan huomioon erytrosyyttien ja plasman välisten veden vaihtoprosessien kesto, punasoluihin sitoutuneen veden tilavuus on otettava huomioon ei-vaihdettavana.

Plasman tilavuus aikuisella on 3,5-5 % kehon painosta. Tälle sektorille on ominaista korkea proteiinipitoisuus, joka määrää vastaavan onkoottisen paineen ja on liikkuvin aineenvaihduntaprosesseissa. Hoidon aikana shokkitilat mistä tahansa syystä tämä ala vaatii erityistä huomiota.

2. Interstitiaalinen sektori sisältää jopa 15 % kehon painosta vettä. Tämän sektorin neste koostuu solujen välisessä tilassa olevasta vedestä ja imusolmukkeesta, joka kiertää kahden puoliläpäisevän kalvon - solu- ja kapillaarikalvon - välillä. Nämä kalvot läpäisevät helposti vettä ja elektrolyyttejä ja vähemmän läpäiseviä plasmaproteiineja. Interstitiaalinen neste on linkki solunsisäisen ja suonensisäisen sektorin välillä, osallistuu homeostaasin ylläpitämiseen, jonka kautta elektrolyytit, happi ja ravinteet pääsevät soluihin ja tapahtuu aineenvaihduntatuotteiden paluuliikettä erityselimiin. Interstitiaalinen neste eroaa veriplasmasta huomattavasti pienemmällä proteiinipitoisuudella. Keho kompensoi akuutin verenhukan ennen kaikkea houkuttelemalla interstitiaalista nestettä verisuonipohjaan. Tämä sektori voi toimia eräänlaisena puskurina. Kun bcc on täydennetty siirtämällä suuria määriä kristalloidiliuoksia, viimeksi mainitut menevät interstitiaaliseen tilaan.

3. Transsellulaarinen sektori on ruoansulatuskanavassa ja muissa suljetuissa onteloissa (esimerkiksi pleuraontelossa) oleva neste. Tämän sektorin tilavuus vaihtelee ajoittain riippuen ruuansulatusnesteiden määrästä, ruuan määrästä ja laadusta, elimistön eritystoimintojen tilasta jne. Yksittäisten kehon osien vesipitoisuus on esitetty kuvassa. 1.

a - suonensisäinen neste,

b - interstitiaalinen neste,

c - solunsisäinen neste.

Homeostaasin ylläpitäminen on mahdollista vain, jos veden saannin ja kehosta erittymisen tiukka tasapaino säilyy. Ensimmäisen ylimäärä toiseen verrattuna normaaleissa olosuhteissa on tyypillistä vain vastasyntyneille (jopa 15-22 ml/vrk) ja alle 1-vuotiaille lapsille (3-5 ml/vrk). Päivittäinen vedentarve aikuiselle

henkilö on 2-3 litraa, mutta tämä arvo tietyistä olosuhteista riippuen (esimerkiksi pitkäaikainen kova fyysinen työ korkea lämpötila ilma), voi nousta jyrkästi ja saavuttaa 10 l/24 tuntia tai enemmän. Lapset kuluttavat enemmän vettä painoyksikköä kohti kuin aikuiset; tämä johtuu heidän kehossaan tapahtuvien redox-prosessien voimakkuudesta.

Vesi pääsee elimistöön juomaveden muodossa (800-1700 ml ja ruoan sisältämä vesi (700-1000 ml); lisäksi kudoksiin muodostuu redox-prosessien aikana noin 200-300 ml vettä. Hyväksyttyjen lisäksi ulkoista nestettä (2 -3 l), ruoansulatusnesteitä liikkuu suuria määriä (jopa 8 l) päivän aikana: sylkeä jopa 1,5 l, mahanestettä 2,5 l, sappia 0,5 l, 0,5 - 0,7 l haimamehua ja 2-3 l suolistomehua. Tämä koko tilavuus (8 l) yhdessä juuri saadun veden (2-3 l) kanssa imeytyy täysin, lukuun ottamatta pientä vesimäärää (150-200 ml) ulosteesta vapautuva. On syytä korostaa, että kaikki veden liikkeet kehossa liittyvät läheisesti elektrolyyttiaineenvaihduntaan. Päivittäinen vedentarve on esitetty taulukossa 1.

Pöytä 1. Päivittäinen vedentarve riippuen

ikä.

Nesteen vapautuminen kehosta tapahtuu munuaisten (enintään 1,5 l), keuhkojen (0,5 l) ja ihon (0,5 l) kautta. Munuaisjärjestelmä säätelee pääasiassa nesteiden koostumusta ja määrää, erittymistä ihon ja keuhkojen kautta ja heijastaa lämmön säätelyn tilaa.

Munuaiset ovat tärkein veden ja elektrolyyttiaineenvaihdunnan säätelyelin kehossa. Päivän aikana jopa 900 litraa verta suodattuu munuaiskuoren glomerulusten läpi; tuloksena olevasta 180 litrasta primääristä ultrasuodosta yli 99 % imeytyy käänteisesti takaisin ja alle 1 % nesteestä erittyy munuaiskuoren muodossa. virtsa. Sen määrä riippuu solunulkoisen nesteen tilavuudesta ja sen sisältämän natriumin määrästä. Mitä enemmän niitä on, sitä voimakkaampi diureesi on. Munuaisten erittymistoiminnan tilan seuranta on yksi avainkohdat erilaisten ääriolosuhteiden hoidossa.

Muista aina, että munuaisten suodatustoiminta pysähtyy, kun paine laskee. renalis jopa 80 mm tai vähemmän. rt. Art., ja jos tämä ajanjakso kestää 1 tunnin tai kauemmin, potilaalle voi kehittyä akuutin munuaisten vajaatoiminnan prerenaalinen muoto.

Normaaliolosuhteissa ihon läpi vapautuu noin 500 ml nestettä vuorokaudessa; jokaisen HS:n kehon lämpötilan nousuun liittyy 500 ml/24 tunnin lisävesihukkaa. Lisääntynyttä hikoilua voidaan havaita collaptoidisissa tiloissa, myrkytyksessä, lämmönsäätelykeskuksen vaurioissa jne. Keho siirtää jopa 20 % lämmöstä hikoilun kautta, mikä selittää hypertermisen oireyhtymän esiintymisen pikkulapsilla liiallisella kääreellä.

Hiki on hypotonista nestettä, joka sisältää liuenneita aineita. Hikirauhasten erityksen elektrolyyttipitoisuus riippuu lisämunuaiskuoren hormonien tasosta: kun niitä ei ole riittävästi, natriumionien vapautuminen hien kautta lisääntyy. Hien natrium- ja klooripitoisuus kasvaa suhteessa hikoilun määrään. Pitkäaikaisessa fyysisessä työssä kuumassa ja kuivassa ilmastossa päivittäinen hikoilu voi ylittää 10 litraa.

Veden erittyminen keuhkojen kautta on keskimäärin 500 ml/24 h. Lihasrasituksessa tai hengenahdistuksessa keuhkojen ventilaatio lisääntyy 3-5 kertaa tai enemmän; Suorassa suhteessa tähän arvoon veden vapautuminen keuhkojen läpi lisääntyy; tässä tapauksessa elektrolyyttien menetystä ei tapahdu.

Kehon eri osien nestemäärän, perifeerisen verenkierron tilan, kapillaarien läpäisevyyden sekä kolloidi-osmoottisten ja hydrostaattinen paine. Tämä suhde on esitetty kaavamaisesti kuvassa. 2

Huomautus:

Nesteeseen vaikuttavan painovoiman aiheuttamaa painetta kutsutaan hydrostatic paine. Se on yhtä suuri kuin nesteen tiheyden painovoiman kiihtyvyyden ja upotussyvyyden tulo.

Osmoottinen on puhtaasta liuottimesta puoliläpäisevällä kalvolla erotettuun liuokseen kohdistuva paine, jossa osmoosi pysähtyy, eli liuotinmolekyylien spontaani tunkeutuminen tämän kalvon läpi, ja riippuu osmoottisesti aktiivisten hiukkasten (ionien ja dissosioitumattomien molekyylien) lukumäärästä. jotka ovat tietyssä tilavuudessa.

Riisi. 2 Nesteen vaihto kapillaarissa.

HD - hydrostaattinen paine;

COP - kolloidinen osmoottinen paine.

Kolloidi-osmoottinen tai onkoottinen, Sitä kutsutaan kolloidisten aineiden aiheuttamaksi liuokseen kohdistuvaksi paineeksi, jonka perustana on albumiini, joka tuottaa noin 80-85 % onkoottisesta paineesta. Normaali plasman onkoottinen paine on noin 25 mmHg. Taide.

Kapillaarin alkuosassa suonensisäinen neste eroaa interstitiaalisesta nesteestä lisääntyneen proteiinipitoisuutensa ja siten korkeamman COD-arvonsa suhteen. Tämä osmoosilakien mukaan (katso edellä) luo olosuhteet nesteen virtaukselle interstitiumista kapillaariin. Samanaikaisesti verenpaine kapillaarin alkuosassa on huomattavasti suurempi kuin interstitiumissa, mikä varmistaa nesteen vapautumisen kapillaarista. Näiden vastasuuntaisten toimien kokonaistulos kapillaarin alkuosassa ilmaistaan ​​ulosvirtauksen vallitsevana ylivirtauksena. Kapillaarin loppuosassa verenpaine laskee ja COP pysyy ennallaan, minkä seurauksena nesteen ulosvirtaus vähenee ja sen sisäänvirtaus hallitsee. Normaaleissa olosuhteissa nesteenvaihtoprosessit verisuonikerroksen ja interstitiaalisen tilan välillä ovat tiukasti tasapainossa.

Patologisissa prosesseissa, jotka liittyvät ensinnäkin plasmassa kiertävän proteiinin häviämiseen (akuutti verenhukka, maksan vajaatoiminta jne.), COP laskee, minkä seurauksena nestettä mikroverenkiertojärjestelmästä alkaa kulkeutua interstitiumiin . Tähän prosessiin liittyy veren paksuuntuminen ja sen reologisten ominaisuuksien rikkominen.

1.2. Elektrolyyttiaineenvaihdunta

Elimistön vesiaineenvaihdunta liittyy läheisesti elektrolyytteihin. Niillä on hallitseva rooli osmoottisessa homeostaasissa. Elektrolyytit otettu Aktiivinen osallistuminen solujen biosähköisen potentiaalin luomisessa, hapen siirtämisessä, energiantuotannossa jne. Näitä aineita löytyy kehon vesisektoreista dissosioituneessa tilassa ioneina: kationeina ja anioneina (katso taulukko 2) . Johtavat kationit solunulkoisessa tilassa (95 %) ovat kalium ja natrium, ja anionit ovat kloridit ja bikarbonaatit (85 %).

Kuten taulukosta 2 voidaan nähdä, vain kalsiumkationit ja bikarbonaattianionit ovat jakautuneet tasaisesti intravaskulaarisissa ja interstitiaalisissa sektoreissa; muiden elektrolyyttien pitoisuudet vaihtelevat varsin merkittävästi niiden toiminnoista riippuen.

Taulukko 2. Elektrolyyttipitoisuus

ihmiskehon vesisektoreilla(keskimääräiset yhteenvetotiedot G. A. Ryabovin, 1982; V. D. Malyshevin, 1985 mukaan).

Huomautus. Vuodesta 1976 lähtien kansainvälisen järjestelmän (SI) mukaisesti aineiden määrä liuoksessa on yleensä ilmaistu millimooleina litrassa (mmol/l). Konsepti "osmolaarisuus" vastaa käsitettä "mo polariteetti" tai "moolipitoisuus". Milliekvivalenteja käytetään, kun ne haluavat heijastaa ratkaisun I sähkövarausta (katso taulukko 3); millimoolia käytetään ilmaisemaan molaarinen pitoisuustavat, eli hiukkasten lukumäärä liuoksessa riippumatta siitä, kuljettavatko ne sähköpurkausta vai ovatko ne neutraaleja; Milliosmolit ovat hyödyllisiä liuoksen osmoottisen vahvuuden osoittamiseen. Pohjimmiltaan konsepti "milliosmole"ja "milimole" biologiset liuokset ovat identtisiä.

Osmolaarisuus liuos ilmaistaan ​​milliosmoleina (mosm) ja se voidaan määrittää litraan vettä liuotettujen erilaisten ionien milliosmoolien (mutta ei milliekvivalenttien) lukumäärän sekä dissosioitumattomien aineiden, kuten glukoosin, urean tai heikosti dissosioituvien aineiden, kuten proteiinin (pitoisuus). joka määrittää yhden onkoottisen paineen komponenteista). Normaalin plasman osmolaarisuus - ledTaso on melko vakio ja on 285-295 mOsmol/l. Plasman pääkomponentti, joka varmistaa sen osmolaarisuuden, on siihen liuenneet natrium- ja kromi-ionit (noin 140 ja 100 mOsmol).

Milliekvivalentti (m/eq)- 1/1000 ekvivalenttia eli kemiallisen alkuaineen määrä, joka yhdistyy paino-osaan vetyä tai korvaa sen. Tämän arvon laskemiseksi sinun on tiedettävä ionimassa ja varausarvo (valenssi).

Mooli (millimooli = 1:1000 mol)- molaarisuusyksikkö, joka vastaa liuosta 1 litrassa, josta 1 mooli ainetta on liuennut.

Esimerkki. 1-molaarinen glukoosiliuos tarkoittaa, että 180 g glukoosia liukenee 1 litraan vettä, mikä vastaa sen yhden molaarista pitoisuutta.

Ihmiskehon joidenkin elinten ja nesteiden emäksisten kationien keskimääräisen pitoisuuden tunteminen (katso taulukko 3) mahdollistaa elektrolyyttiaineenvaihdunnan häiriöiden oikean arvioinnin erilaisissa patologioissa.

NATRIUM on tärkein kationi interstitiaalisessa tilassa (katso taulukko 2).

Taulukko 3. Pääkationien keskimääräinen pitoisuusjoissakin ihmiskehon elimissä ja nesteissä (mmol/l)

Kun sen pitoisuus pienenee, osmoottinen paine laskee samalla, kun interstitiaalisen tilan tilavuus pienenee; sen pitoisuuden lisääntyminen aiheuttaa päinvastaisen prosessin. Natriumin puutetta ei voida kompensoida millään muulla kationilla. Plasman puutteen ja natriumin puutteen välillä on lineaarinen suhde (Gregersen J., 1971). Aikuisen natriumin päivittäinen tarve on 5-10 g.

Natrium erittyy elimistöstä pääasiassa munuaisten kautta; pieni osa erittyy hien mukana. Sen taso veressä nousee pitkäaikaisessa kortikosteroidihoidossa, pitkäaikaisessa mekaanisessa ventilaatiossa hyperventilaatiotilassa, diabetes insipiduksessa, hyperaldosteronismissa ja laskee diureettien pitkäaikaisen käytön vuoksi, pitkäaikaisen hepariinihoidon taustalla, kroonisen sydämen vajaatoiminnan, hyperglykemian, maksakirroosi jne.

Huomautus. 1 mekv natriumia = 1 mmol = 23 mg; 1 g natriumia = 43,5 mmol.

HYPERNATRAEMIA (plasman natrium yli 147 mmol/l) ilmenee, kun interstitiaalisen tilan natriumpitoisuus on kohonnut. Siihen liittyy nesteen uudelleenjakautuminen solunsisäisestä solunulkoiseen sektoriin, mikä aiheuttaa solujen kuivumista. SISÄÄN hoitokäytäntö tämä tila voi johtua lisääntynyt hikoilu, hypertonisen natriumkloridiliuoksen suonensisäinen infuusio, sekä akuutin munuaisten vajaatoiminnan kehittymisen yhteydessä.

HYPONATREMIA (plasmanatrium alle 137 mmol/l) kehittyy liiallisen ADH:n erittymisen seurauksena kiputekijän vaikutuksesta, maha-suolikanavan patologisista häviöistä, suolattomien liuosten tai glukoosiliuosten liiallisesta suonensisäisestä annosta ja siihen liittyy solujen liiallinen nesteytys. samanaikainen BCC:n lasku.

KALIUM on tärkein solunsisäinen kationi (katso taulukko 2). 98 % tästä elektrolyytistä löytyy eri elinten ja kudosten soluista. Aikuisen ihmisen päivittäinen kaliumtarve on 60-80 mmol (2,3-3,1 g). Tämä elektrolyytti osallistuu aktiivisesti kaikkiin kehon aineenvaihduntaprosesseihin; sen aineenvaihdunta liittyy läheisesti natriumiin. Kaliumilla, kuten natriumilla, on johtava rooli kalvopotentiaalien muodostumisessa; se vaikuttaa pH-arvoon ja glukoosin käyttöön.

Huomautus. 1 g kaliumia = 25,6 mmol; 1 g KCI:tä sisältää 13,4 mmol K; 1 mEq kaliumia = 1 mmol = 39,1 mg.

HYPOKALEMIA (plasman kalium alle 3,8 mmol/l) voi kehittyä natriumylimäärällä, metabolisen alkaloosin taustalla, hypoksialla, vakavalla proteiinikatabolialla, ripulilla, pitkittyneellä oksentelulla jne. Solunsisäisen kaliumin puutteen yhteydessä natriumioneja alkaa tulla solu intensiivisesti ja vety; se aiheuttaa kehitystä solunsisäinen asidoosi ja ylihydraatio solunulkoisen metabolisen alkaloosin taustalla. Kliinisesti tämä tila ilmenee sydämen rytmihäiriöinä, hypotensiona, poikkijuovaisten lihasten heikentyneenä sävynä, suoliston pareesina ja mielenterveyshäiriöinä. EKG:ssä näkyy tyypillisiä muutoksia: takykardia, kapeneminen QRS-kompleksi, T-aallon väheneminen.

Hypokalemian hoito alkaa etiologisen tekijän poistamisella, jota seuraa kaliumvajeen kompensointi, joka määritetään kaavalla:

Kaliumin puute (mmol/l) = (5,0 - K potilaan plasmasta mmol/l) 0,2ruumiinpaino kg.

Suurien kaliumlisämäärien nopea antaminen voi aiheuttaa sydänkomplikaatioita, mukaan lukien sydämenpysähdyksen, Siksi vuorokausiannos ei saa olla yli 3 mmol/kg/vrk ja infuusionopeus enintään 20 mmol/h.

On suositeltavaa laimentaa käytetyt kaliumvalmisteet 40 mmol:iin injektioliuosta litraa kohti; on optimaalista antaa ne polarisoivan seoksen muodossa (glukoosi - kalium - insuliini). Käsittely kaliumvalmisteilla on suoritettava päivittäisessä laboratoriovalvonnassa.

Esimerkki. 70 kg painavalla potilaalla on taudista johtuva vaikea hypokalemia (plasman kalium 3,2 mmol/l). Ottaen huomioon yllä olevat tiedot, että kaliumin vuorokausiannos ei saisi olla yli 3 mmol/kg/vrk, laskemme suurimman vuorokausiannoksen: osoittautuu 210 mmol/70 kg/24 tuntia ja laskimonsisäisen annostelun aika. tämän kaliummäärän tulee olla vähintään 10,5 tuntia (210:20).

HYPERKALEMIA (plasman kalium yli 5,2 mmol/l) esiintyy useimmiten, kun kaliumin erittyminen kehosta (OPN) on häiriintynyt tai kun tämä elektrolyytti vapautuu massiivisesti vaurioituneista soluista: trauma, punaisen veren hemolyysi Lisäksi tämän oireyhtymän esiintyminen on mahdollista hypertermian, kouristusoireyhtymän ja tiettyjen lääkkeiden käytön yhteydessä: hepariini, aminokapronihappo, mannitoli ja monet muut.

Hyperkalemian diagnoosi perustuu etiologisten tekijöiden esiintymiseen (trauma, akuutti munuaisten vajaatoiminta jne.), tyypillisten sydämentoiminnan muutosten ilmaantumiseen: sinusbradykardia (sydämenpysähdykseen asti) yhdistettynä kammion ekstrasystolaan ja ominaisiin laboratoriotietoihin (plasma). kalium yli 5,5 mmol/l). EKG:ssä näkyy korkea, terävä T-aalto ja QRS-kompleksin laajeneminen.

Hyperkalemian hoito alkaa etiologisen tekijän poistamisella ja asidoosin korjaamisella. Kalsiumvalmisteita käytetään antagonisteina; hyvä lääke siirtääksesi ylimääräisen plasman kaliumin soluun, käytä glukoosiliuosta (10-15 %) ja insuliinia (1 yksikkö jokaista 3-4 g glukoosia kohden). Jos nämä menetelmät eivät tuota toivottua vaikutusta, hemodialyysi on aiheellinen.

KALSIUM (katso taulukko 2) muodostaa noin 2 % kehon painosta, josta 99 % on sidottu tila luissa ja normaaleissa olosuhteissa ne eivät osallistu elektrolyyttiaineenvaihduntaan. Kalsiumista on liuennut noin 1 %, 50-60 %

ionisoitua tästä arvosta. Tämä kalsiumin muoto osallistuu aktiivisesti impulssien hermo-lihasvälitykseen, veren hyytymisprosesseihin, sydänlihaksen työhön ja sähköpotentiaalin muodostukseen. solukalvot ja useiden entsyymien tuotanto. Päivittäinen tarve on 700-800 mg. Tämä mikroelementti tulee kehoon ruoan mukana, erittyy maha-suolikanavan kautta ja virtsaan. Kalsiumin aineenvaihdunta liittyy läheisesti fosforin aineenvaihduntaan, plasman proteiinitasoihin ja veren pH:hon.

Huomautus. 1 meq kalsiumia = 0,5 mmol, 1 mmol = 40 mg, 1 g = 25 mmol.

HYPOKALSIEMIA (plasman kalsium alle 2,1 mmol/l) kehittyy hypoalbuminemiaan, haimatulehdukseen, suurten sitraattiveren siirtoon, pitkäaikaisiin sappifisteleihin, D-vitamiinin puutteeseen, imeytymishäiriöön ohutsuolessa, traumaattisten leikkausten jälkeen jne. Kliinisesti tämä ilmenee lisääntynyttä hermo-lihashermoisuutta, parestesian ilmaantumista, kohtauksellista takykardiaa, tetaniaa. Hypokalsemia korjataan sen jälkeen, kun sen taso veriplasmassa on määritetty laboratoriossa antamalla suonensisäisesti ionisoitua kalsiumia sisältäviä lääkkeitä: glukonaattia, laktaattia, kalsiumkloridia tai karbonaattia, kaikkia näitä toimintoja ei kuitenkaan olevaikutus ilman albumiinitasojen ennalta normalisoitumista.

HYPERKALSIEMIA (plasman kalsium yli 2,6 mmol/l) esiintyy kaikissa prosesseissa, joihin liittyy lisääntynyt luutuho (kasvaimet, osteomyeliitti), lisäkilpirauhasen sairaudet (adenooma tai lisäkilpirauhastulehdus), liiallinen kalsiumlisän antaminen sitraattiverensiirron jälkeen jne. Kliinisesti tämä tila alkaa ilmetä lisääntyneenä väsymyksenä, letargiana, lihas heikkous. Kun hyperkalsemia lisääntyy, ilmaantuu maha-suolikanavan atonian oireita: pahoinvointia, oksentelua, ummetusta, ilmavaivat. EKG:ssä näkyy tyypillinen ST-välin lyheneminen.

Hoito koostuu patogeneettiseen tekijään vaikuttamisesta. Vaikeassa hyperkalsemiassa (yli 3,75 mmol/l) tarvitaan kohdennettua korjausta. Tätä tarkoitusta varten suositellaan 2 g:n dinatriumsuolaa, joka on laimennettuna 500 ml:aan 5-prosenttista glukoosiliuosta, laimennettuna tyleenidiamiinitetraetikkahapon (EDTA). Tämä lääke tulee antaa suonensisäisesti, hitaasti, tiputtaen, 2-4 kertaa päivässä, veriplasman kalsiumpitoisuuden hallinnassa.

MAGNESIUM on solunsisäinen kationi. Sen pitoisuus plasmassa on 2,15 kertaa pienempi kuin punasoluissa (ks. taulukko 3). Tällä mikroelementillä on estävä vaikutus hermo-lihasjärjestelmän kiihtyvyyteen ja sydänlihaksen supistumiskykyyn, mikä aiheuttaa keskushermoston lamaa. Magnesiumilla on tärkeä rooli entsymaattisissa prosesseissa: hapen imeytymisessä, energiantuotannossa jne. Se pääsee elimistöön ruoan mukana ja erittyy maha-suolikanavan ja virtsan kautta.

Huomautus. 1 meq magnesiumia = 0,5 mmol. 1 mmol = 24,4 mg. 1 f = 41 mmol.

HYPOMAGNEMIAA (plasman magnesium alle 0,8 mmol/l) havaitaan maksakirroosissa, kroonisessa alkoholismissa, akuutissa haimatulehduksessa, polyuriassa akuutin munuaisten vajaatoiminnan vaiheet, suoliston fistelit, epätasapainoinen infuusiohoito jne. Kliinisesti tämä tila ilmenee lisääntyneenä hermo-lihashermoitumisena, hyperrefleksiana, eri lihasryhmien kouristuksina; Spastista maha-suolikanavan kipua, oksentelua ja ripulia voi esiintyä. Hoito koostuu kohdistetusta vaikutuksesta etiologiseen tekijään ja magnesiumsuolojen antamiseen laboratoriovalvonnassa.

HYPERMAGNEMIA (plasman magnesium yli 1,2 mmol/l) kehittyy ketoasidoosin, lisääntyneen katabolian, akuutin munuaisten vajaatoiminnan yhteydessä. Kliinisesti ilmenee uneliaisuuden ja letargian, hypotension ja bradykardian kehittymisenä, hengityksen heikkenemisenä ja hypoventilaatioiden ilmaantuessa. Hoito koostuu kohdistetusta vaikutuksesta etiologiseen tekijään ja kemiallisen magnesiumantagonistin - kalsiumsuolojen - määräämisestä.

KLOORI on tärkein anioni solunulkoisessa tilassa (katso taulukko 2). Sen tasoa säätelee aldosteroni. Klooria löytyy natriumia vastaavassa suhteessa. Klorideilla on osmoottinen vaikutus veteen, eli se menee sinne, missä on kloorianioneja. Kloori-ionit tulevat kehoon muodossa natriumkloridia, jälkimmäinen hajoaa mahassa natriumkationeiksi ja kloorianioneiksi. Tämän jälkeen natrium muodostaa natriumbikarbonaattia, ja kloori yhdistyy vedyn kanssa muodostaen suolahappoa.

Huomautus. 1 mekv klooria = 1 mmol = 35,5 mg. 1 g klooria = 28,2 mmol.

HYPOKLOREAEMINEN SYNDROOMI (plasman kloori alle 95 mmol/l) kehittyy pitkittyneen oksentamisen, vatsakalvontulehduksen, pylorisen ahtauman, korkean suolitukoksen ja lisääntyneen hikoilun yhteydessä.

Tämän oireyhtymän kehittymiseen liittyy bikarbonaattipuskurin lisääntyminen ja alkaloosin ilmaantuminen. Kliinisesti tämä tila ilmenee kuivumisena, hengitys- ja sydänongelmina. Mahdollinen kouristuksen esiintyminen tai koomaan tilaan kohtalokkaalla seurauksella. Hoito koostuu kohdistetusta vaikutuksesta patogeneettiseen tekijään ja infuusiohoidosta klorideilla (pääasiassa natriumkloridivalmisteilla) laboratoriovalvonnassa.

HYPERKLOREMIA (plasman kloori yli 110 mmol/l) kehittyy yleisen nestehukkauksen, nesteen poistumisen heikkenemisen yhteydessä interstitiaalisesta tilasta (esim. akuutti munuaisten vajaatoiminta), lisääntyneeseen nesteen siirtymiseen verisuonikerroksesta interstitiumiin (hypoproteinemiaan) ja suurten nestemäärien lisääminen, jotka sisältävät ylimääräistä klooria. Tämän oireyhtymän kehittymiseen liittyy veren puskurikapasiteetin väheneminen ja metabolisen asidoosin ilmaantuminen.

Kliinisesti tämä tila ilmenee turvotusoireyhtymän kehittymisenä (esimerkiksi interstitiaalinen keuhkopöhö). Hoidon pääperiaate on vaikutus patogeneettiseen tekijään yhdistettynä syndroomahoitoon.

BIKBONAATIANIONIT ovat osa tärkeintä solunulkoisen tilan bikarbonaattipuskuria. Tämän puskurin pitoisuutta kehossa säätelee munuaisjärjestelmä, ja se riippuu suurelta osin kloorin ja useiden muiden ionien pitoisuudesta. Valtimoveressä bikarbonaattipuskurin taso on 18-23 mmol/l, valtimoveriplasmassa 1 21-28 mmol/l, laskimoveressä 22-29 mmol/l. Sen pitoisuuden laskuun liittyy metabolisen asidoosin kehittyminen, ja lisääntyminen aiheuttaa metabolisen alkaloosin ilmaantumisen.

Happo-emästila.

Huomautus. 1 meq bikarbonaattia = 1 mmol = 80 mg.

1 g bikarbonaattia = 12,5 mmol.

FOSFAATTI on tärkein anioni solunsisäisessä tilassa (katso taulukko 2). Plasmassa sitä esiintyy monovetyfosfaattina ja vetyfosfaattianioneina (normaali: 0,65-1,3 mmol/l).

Fosfaattien päätehtävä on osallistua energian aineenvaihduntaan; Lisäksi ne osallistuvat aktiivisesti proteiinien ja hiilihydraattien aineenvaihduntaan. Tämän mikroelementin puutos voi ilmetä akuutin munuaisten vajaatoiminnan ja pitkittyneen paaston, sepsiksen, kroonisten maha-suolikanavan sairauksien, pitkäaikaisen steroidihormonihoidon, asidoosin jne. yhteydessä.

Kliinisesti tämä tila ilmenee nopeana väsymyksenä, heikkoutena, adynamiana ja hyporefleksiana. Hoito on patogeneettistä.

3. Vesi-elektrolyyttiaineenvaihdunnan häiriöiden päätyypit

Kun veden virtaus kehoon on rajoitettua tai sen jakautuminen kehossa häiriintyy, tapahtuu kuivumista. Nesteen puutteesta riippuen erotetaan lievä, keskivaikea ja vaikea kuivumisaste. Lievää nestehukkaa esiintyy, kun kehon nestehukka on 5-6 % (1-2 l), kohtalainen - 5-10 % (2-4 l) ja vaikea - yli 10 % (yli 4-5 l) . Akuutti 20 prosentin tai enemmän nesteen menetys kehosta on kohtalokasta.

3.1. Menetelmät kuivumisasteen määrittämiseksi

Lievä kuivuminen ilmenee kliinisesti janon ja kuivuuden ilmaantumisena suuontelon suun kautta hengitettäessä (esimerkiksi potilaalla on letku vatsaan nenän kautta) havaitaan suun limakalvon kuivumista myös nesteen puutteen puuttuessa. Tässä tilanteessa sinun on tarkistettava kainalon tai nivusalueiden ihon kunto. Normaaleissa olosuhteissa iho on aina kostea. Kuivuuden esiintyminen viittaa ainakin lievään kuivumiseen. Kuivumisasteen selvittämiseksi voit käyttää melko yksinkertaista testiä: 0,25 ml ruiskutetaan intradermaalisesti kyynärvarren etupinnan alueelle. suolaliuosta natriumkloridi ja merkitse aika injektiohetkestä täydelliseen resorptioon ja läpipainopakkauksen katoamiseen (normi on 45-60 minuuttia). Ensimmäisellä dehydraatioasteella resorptioaika on 30-40 minuuttia. sen korjaamiseksi tarvitaan nesteitä 50-80 ml/kg/24 tuntia; toisessa asteessa - 15-20 minuuttia. ja 80-120 ml/kg/24 tuntia, ja kolmannessa asteessa - 5-15 minuuttia. ja 120-169 ml/kg/24 tuntia.

Kliinisiin oireisiin, joiden avulla voidaan määrittää kehon nesteen epätasapainon tyyppi, mukaan lukien jano, kunto iho ja näkyvät limakalvot, kehon lämpötila, yleinen tila potilas ja hänen neurologinen ja henkinen tila, turvotuksen esiintyminen, keskeiset hemodynaamiset indikaattorit: verenpaine, keskuslaskimopaine, syke, hengitystila, diureesi, laboratoriotiedot.

Vesi-, elektrolyytti- ja happo-emästasapainon häiriöiden välillä on läheinen yhteys. Patologiset menetykset tai riittämätön veden saanti ja erittyminen kehosta vaikuttavat ensisijaisesti interstitiaaliseen sektoriin. Keho sietää kuivumista paljon vaikeammin kuin ylinestystä. Esimerkki tästä tilanteesta olisi ketoasidoottinen kooma - kuolema ei tapahdu niinkään kehon myrkytyksen seurauksena, vaan aivosolujen kuivumisen seurauksena.

Kokeilu osoitti, että 20-30 % välitilan tilavuuden nopea menetys on kohtalokasta, mutta samalla sen kasvua, jopa kaksinkertaistamista, siedetään varsin tyydyttävästi. Interstitiaalisen nesteen osmoottinen pitoisuus määräytyy siinä olevien natriumionien pitoisuuden mukaan. Sen tasosta riippuen ne erottavat isotoninen(natrium on normaalia) hypotoninen(natrium alle normaalin) ja hypertensiivinen(natrium yli normaalin) dehydirotaatio ja ylihydraatio.

Kuivumisen tyypit

3.2. Isotoninen nestehukka

Isotoninen dehydraatio (plasmanatrium normaalialueella: 1 135-145 mmol/l) johtuu nesteen menetyksestä interstitiaalisessa tilassa, joka on elektrolyyttikoostumukseltaan lähellä veriplasmaa, eli patologian tyyppi huomioon ottaen on olemassa tasainen nesteen ja natriumin menetys. Useimmiten tämä patologinen tila ilmenee pitkittyneellä oksennuksella ja ripulilla, akuutilla ja krooniset sairaudet Ruoansulatuskanava, suolen tukkeuma, vatsakalvontulehdus, haimatulehdus, laajat palovammat, polyuria, kontrolloimaton diureettien antaminen, polytrauma jne. Kuivumiseen liittyy elektrolyyttihäviö ilman merkittävää muutosta plasman osmolaarisuus, joten merkittävä veden uudelleenjakautuminen sektorien välillä ei esiintyy, mutta hypovolemia muodostuu.

Kliinisesti havaitaan häiriöitä keskushemodynamiikassa: verenpaine, keskuslaskimopaine ja verenpaineen lasku. Ihon turgor pienenee, kieli kuivuu, kehittyy oliguriaa tai jopa anuriaa.

Hoito koostuu kohdistetusta vaikutuksesta patogeneettiseen tekijään ja korvaushoidosta isotonisella natriumkloridiliuoksella (35-70 ml/kg/vrk). Infuusiohoito tulee suorittaa keskuslaskimopaineen ja tunnin välein tapahtuvan diureesin hallinnassa.

3.3. Hypotoninen nestehukka

Hypotoninen dehydraatio (plasman natrium alle 130 mmol/l) kehittyy tapauksissa, joissa natriumin hävikki on suurempi kuin veden hävikki. Tämä oireyhtymä ilmenee sisältävien nesteiden massiivisten hävikkien yhteydessä suuri määrä elektrolyytit: toistuva oksentelu, runsas ripuli, runsas hikoilu, polyuria. Veriplasman natriumpitoisuuden laskuun liittyy sen osmolaarisuuden heikkeneminen, minkä seurauksena plasman vesi alkaa jakautua uudelleen soluihin, mikä aiheuttaa niiden turvotusta (sellunsisäistä hyperhydraatiota) ja syventää nesteen puutteen ilmiötä välitila.

Kliinisesti Tämä tila ilmenee ihon turgorin vähenemisenä ja silmämunat, verenkiertohäiriöitä, atsotemiaa, munuaisten ja aivojen vajaatoimintaa ja veren paksuuntumista ilmenee. Hoito koostuu kohdistetusta vaikutuksesta patogeneettiseen tekijään ja kehon aktiivisesta nesteytyksestä natriumkationeja sisältävillä lääkkeillä. Jälkimmäinen lasketaan kaavalla:

PuuteNa + (mmol/l) = (142 mmol/l -Napotilaan plasmammol/l) 0,2 ruumiinpainoa (kg)

Jos hypotonisen dehydraation korjaus suoritetaan metabolisen asidoosin taustalla, natriumia annetaan bikarbonaatin muodossa, metabolisen alkaloosin tapauksessa - kloridin muodossa.

3.4. Hypertensiivinen nestehukka

Hypertoninen dehydraatio (plasman natrium yli 150 mmol/l) tapahtuu, kun vesihäviö ylittää natriumhäviön.

Tämä tila esiintyy akuutin munuaisten vajaatoiminnan polyuriavaiheessa, pitkittyneessä pakkodiureesissa ilman veden puutteen oikea-aikaista täydentämistä, kuumetta, riittämättömän veden antamisen aikana. parenteraalinen ravitsemus. Ylimääräinen veden menetys natriumiin nähden lisää plasman osmolaarisuutta, minkä seurauksena solunsisäistä nestettä alkaa kulkeutua verisuonikerrokseen. Muodostuu solunsisäinen dehydraatio (solujen eksikoosi).

Kliinisesti Tämä tila ilmenee jano, heikkous, apatia. Aivosolujen kuivuminen aiheuttaa epäspesifisten neurologisten oireiden ilmaantumista: psykomotorista levottomuutta, sekavuutta, kouristuksia, kooman kehittymistä. On kuivaa ihoa, kohonnutta ruumiinlämpöä, oliguriaa, johon liittyy tiivistetyn virtsan vapautumista, ja veren paksuuntumista. Hoito koostuu kohdistetusta vaikutuksesta patogeneettiseen tekijään ja solunsisäisen kuivumisen poistamiseen määräämällä glukoosiliuosta infuusioina insuliinin kanssa.

Hyperhydraation tyypit

3.5. Isotoninen hyperhydraatio

Isotoninen ylihydraatio (plasmanatrium normaalirajoissa: 135-145 mmol/l) esiintyy useimmiten sellaisten sairauksien taustalla, joihin liittyy turvotusoireyhtymä (krooninen sydämen vajaatoiminta, raskauden toksikoosi), isotonisten suolaliuosten liiallisen antamisen seurauksena. Tämän oireyhtymän esiintyminen on mahdollista myös maksakirroosin ja munuaissairauksien (nefroosi, glomerulonefriitti) taustalla.

Isotonisen ylihydraation kehittymisen perusta on interstitiaalisen nesteen määrän lisääntyminen natriumin ja veden suhteellisen pidättymisen taustalla kehossa. Plasman osmoottinen paine ei muutu.

Kliinisesti tämä hyperhydraation muoto ilmenee ulkonäöstä hypertensio, kehon painon nopea nousu, turvotusoireyhtymän kehittyminen, anasarca, veren pitoisuusparametrien lasku. Ylihydraation taustalla kehossa on vapaan nesteen puute - tämä aiheuttaa janoa.

Tämän patologian hoito koostuu patogeneettiseen tekijään kohdistuvan kohdistetun toiminnan lisäksi käytöstä hoitovaihtoehdot tarkoituksena on vähentää välitilan määrää. Tätä tarkoitusta varten annetaan suonensisäisesti 10 % albumiinia (se lisää plasman onkoottista painetta, minkä seurauksena interstitiaalinen neste alkaa kulkeutua verisuonikerrokseen) ja diureetteja. Jos tämä hoito ei anna toivottua vaikutusta, voidaan käyttää hemodialyysia veren ultrasuodatuksella.

3.6. Hyperhydraatio hypotoninen

Hypotoninen ylihydraatio (plasman natrium alle 130 mmol/l) tai " vesimyrkytys", voi tapahtua, kun nautit samanaikaisesti hyvin suuria vesimääriä (esimerkiksi henkilö oli autiomaassa pitkään ilman vettä ja joi sitten välittömästi jopa 10 litraa tai enemmän vettä), kun suolattomat liuokset, kroonisesta sydämen vajaatoiminnasta johtuva turvotus, maksakirroosi, akuutti munuaisten vajaatoiminta, ADH:n ylituotanto jne. Tässä patologisessa tilassa plasman osmolaarisuus laskee ja vettä alkaa tunkeutua soluihin, mikä aiheuttaa neurologisten oireiden ilmaantumista ( syynä on aivoturvotus).

Kliinisesti Tämä tila ilmenee oksentamisena, usein löysänä, vetisenä ulosteena ja polyuriana. Lisätään merkkejä keskushermostovauriosta: heikkous, heikkous, väsymys, unihäiriöt, delirium, tajunnan heikkeneminen, kouristukset, kooma. Hoito koostuu patogeneettiseen tekijään kohdistuvan kohdennetun toiminnan lisäksi ylimääräisen veden poistamisesta elimistöstä mahdollisimman nopeasti. Tätä tarkoitusta varten määrätään diureetteja; hemodialyysiä veren ultrasuodatuksella voidaan käyttää.

3.7. Hyperhydraatio hypertensiivinen

Hypertonista ylihydraatiota (plasmanatrium yli 150 mmol/l) esiintyy, kun elimistöön annetaan suuria määriä hypertonisia liuoksia, joiden munuaisten erittymistoiminto on säilynyt, tai isotonisia liuoksia potilaille, joilla on munuaisten eritystoiminnan vajaatoiminta. Tähän tilaan liittyy nesteen osmolaarisuuden lisääntyminen interstitiaalisessa tilassa, jota seuraa solusektorin kuivuminen ja lisääntynyt kaliumin vapautuminen siitä.

Kliiniseen kuvaan Tälle hyperhydraation muodolle on ominaista jano, ihon punoitus, kohonnut ruumiinlämpö, ​​verenpaine ja keskuslaskimopaine. Prosessin edetessä ilmenee merkkejä keskushermostovauriosta: mielenterveyshäiriöt, kouristukset, kooma.

Hoito koostuu etiologiseen tekijään kohdistuvan vaikutuksen lisäksi infuusiohoito natiivi suolaliuosten korvaaminen proteiineilla ja glukoosiliuoksilla sekä osmodiureettien ja salureettien käyttö. Vaikeissa tapauksissa hemodialyysi on aiheellinen.

4. Vesitasapainon laskeminen

Normaalioloissa veden saanti elimistöön on yhtä suuri kuin sen erittyminen. Vesitasetta laskettaessa on otettava huomioon seuraavat seikat:

    Sisäänpääsy: enteraalinen, parenteraalinen ja endogeeninen vesi (200-300 ml/24 tuntia).

    Fysiologiset menetykset: päivittäinen diureesi, erittyminen keuhkojen (500 ml/24 tuntia), ihon (500 ml/24 tuntia) kautta ja ulosteiden hävikki - 150-200 ml. Kun lämpötila nousee yli 37 °C, lisää 500 ml jokaista HS:ää kohti.

    Patologiset menetykset: oksentelu, ripuli, fistelit, vedenpoisto, aspiraatio.

Kehon vedentarvetta laskettaessa lähdetään keskiarvosta: 35-40 ml/1 painokilo/24 tuntia.

Vesitase lasketaan potilaille päivässä tiettynä aikana. Jos syötettävän nesteen määrä vastaa häviöitä, tämä tulkitaan vesitaseeksi nolla, häviöiden ylitys on positiivinen ja vähemmän kuin hävikki negatiivinen.

Kaavat vesitasapainon laskemiseen

V 1 = (m40) + (k500) – x 1 (x 2 );

V 2 = (14,5 m) + (k 500) + d - 200;

VA = V 1 - V2;

missä: V 1 - elimistön vedentarve ml/24 tuntia,

V 2 - annosteluun tarvittavan vesimäärän laskeminen ml/24 tuntia,

V A - vesitase päivässä,

m - massa kg,

k - lämpötilakerroin,

k - (potilaan t°C - 37);

k = 0 potilaan t 37 °C:ssa ja sen alapuolella;

k = 1 potilaan 38 °C:ssa ja sitä korkeammassa lämpötilassa;

k = 2 potilaan t 39 °C:ssa ja sitä korkeammassa lämpötilassa.

X 1 = 300 (aikuisille) - endogeenisen veden määrä;

x 2 = 150 (lapsille) - endogeenisen veden määrä;

d - diureesi.

Jos VA = 0 - nolla vesitase,

Kun A >0 - positiivinen vesitase, V A<0 - отрицательный водный баланс.

5. Elektrolyyttivajeen laskeminen

JA VAADITTUJEN RATKAISIEN MÄÄRÄ

NIIDEN KORJAUKSEEN

Elektrolyyttivajeen ja niiden korjaamiseen tarvittavien liuosten määrän laskemiseksi sinun on tiedettävä merkittävimpien kemiallisten yhdisteiden ekvivalenttisuhteet:

Natriumkalium

1 mekv. = 1 mmol = 23,0 mg 1 mekv. = 1 mmol = 39,1 mg 1 g = 43,5 mmol 1 g = 25,6 mmol

KalsiumMagnesium

1 mekv. = 0,5 mmol 1 mekv. = 0,5 mmol

1 mmol = 40,0 mg 1 mmol - 24,4 mg

1 g = 25 mmol 1 g = 41 mmol

KlooriHiilikarbonaatti

1 mekv. = 1 mmol = 35,5 mg 1 mekv. = 1 mmol = 61,0 mg 1 g = 28,2 mmol 1 g = 16,4 mmol

Natriumkloridia

1 g NaCl:a sisältää 17,1 mmol natriumia ja 17,1 mmol klooria. 58 mg NaCl:a sisältää 1 mmol natriumia ja 1 mmol klooria. 1 litra 5,8 % NaCl-liuosta sisältää 1000 mmol natriumia ja 1000 mmol klooria.

1 g NaCl:a sisältää 400 mg natriumia ja 600 mg klooria.

kaliumkloridi

1 g KC1 sisältää 13,4 mmol kaliumia ja 13,4 mmol klooria.

74,9 mg KC1:tä sisältää 1 mmol kaliumia ja 1 mmol klooria.

1 litra 7,49 % KS1-liuosta sisältää 1000 mmol kaliumia ja 1000 mmol klooria.

1 g KCl:a sisältää 520 mg kaliumia ja 480 mg klooria.

Natriumbikarbonaatti

1 g natriumbikarbonaattia (NaHCO 3) sisältää 11,9 mmol natriumia ja 11,9 mmol bikarbonaattia.

84 mg NaHC03:a sisältää 1 mmol natriumia ja 1 mmol bikarbonaattia.

1 litra 8,4 % NaHCO 3 -liuosta sisältää 1000 mmol natriumia ja; 1000 mmol bikarbonaattia.

Kaliumbikarbonaatti

1 g KHC03:a sisältää 10 mmol kaliumia ja 10 mmol bikarbonaattia.

Natriumlaktaatti

1 g NaC 3 H 5 O 2 sisältää 8,9 mmol natriumia ja 8,9 mmol laktaattia. Minkä tahansa elektrolyytin puute mmol/l voidaan laskea yleiskaavalla:

Elektrolyyttivaje (D) (mmol/l) = (K 1 K 2 ) potilaan paino 0,2

Huomautus: K 1 - normaali anionien tai kationien pitoisuus plasmassa, mmol/l; K 2 - anionien tai kationien pitoisuus potilaan plasmassa, mmol/l.

Annostukseen tarvittava elektrolyyttiliuoksen määrä (V) ml:na lasketaan korjausta varten kaavalla:

V= AD (elektrolyyttivaje, mmol/l),

missä A on kerroin (tiedon liuoksen määrä, joka sisältää

1 mmol anionia tai kationia):

3 % liuos KS1 - 2,4 10 % liuos CaCl -1,1

7,5 % liuos KS1 - 1,0 2 % liuos NS1 - 1,82

10 % NaCl-liuos - 0,58 5 % NaHC03-liuos - 1,67

5,8 % NaCl-liuos - 1,0 10 % Na-laktaattiliuos - 1,14

5 % liuos NH 4 C1 - 1,08 25 % liuos MgSO 4 - 0,5

5,4 NH 4 C1 -liuos - 1,0 0,85 % NaCl-liuos - 7,1

Esimerkki. 70 kg painavalla potilaalla on vakava hypokalemia (plasman kalium 3,0 mmol/l) perussairauden taustalla. Yllä olevan kaavan avulla määritämme elektrolyytin puutteen:

D (mmol/l) = 70 (potilaan paino) 0,2 (5,0 - 3,0)

Plasman kaliumvaje tällä potilaalla on 28 mmol (1 mmol = 39,1 mg (katso edellä), joten grammoina se on 39,1 mg - 28 mmol = 1,095 g) Seuraavaksi lasketaan elektrolyyttiliuoksen määrä (V ) ml:na tarvitaan antoa varten korjausta varten. Käytämme elektrolyyttinä 3 % CS-liuosta! (Katso edellä).

V= A D = 2,4 28 = 67,2 ml

Tämä elektrolyyttitilavuus (67,2 ml) on laimennettava 40 mmol:iin litrassa 5-10 % glukoosilla ja annettava laskimoon polarisoivana seoksena. Ottaen huomioon, että kaliuminfuusion nopeus ei saa olla yli 20 mmol/h, määritämme 28 mmol:n kaliumin (67,2 ml 3 % KS1:n) antamisen vähimmäiskestoksi, joka on noin 1,5 tuntia (90 minuuttia).

On olemassa muita laskentakaavoja, joiden avulla voit määrittää välittömästi tarvittavan tilavuuden standardiliuoksia elektrolyyttiaineenvaihdunnan häiriöiden korjaamiseen (A.P. Zilber, 1982):

solunulkoisen kaliumin puutos:

3 % KC1 (ml) = 0,5 massa (kg) (5-K plasma (pl.);

joilla on solunsisäinen kaliumin puutos:

3 % KS1 (ml) = massa (kg) (115 - K erytrosyyttejä (er.); solunulkoisen kalsiumin puutos:

10 % CaCl 2 (ml) = 0,11 massa (kg) (0,5 - Ca pl.);

solunsisäinen kalsiumin puutos:

10 % CaCl 2 (ml) = 0,22 massa (kg) (0,75 - Caer.);

solunulkoisen natriumin puutteen kanssa:

10 % NaCl (ml) = 0,12 - massa (kg) (142 - Na pl.); solunsisäinen natriumin puute:

10 % NaCl (ml) = 0,23 massa (kg) (20 - Naer.); solunulkoisen magnesiumin puutos:

25 % MgS04 (ml) = 0,05 massa (kg) (2,5 - Mg pl.); solunsisäinen magnesiumin puutos:

25% MgSO 4 (ml) = 0,1 - massa (kg) (5,2 -Mger.).

Huomautus. Elektrolyyttivajeen korjaaminen tulee aloittaa kationista tai anionista, jonka puutos on vähemmän ilmeinen.

6. Plasman osmolaarisuuden laskeminen

Plasman osmolaarisuus määritetään erityisillä laboratorioinstrumenteilla, mutta niiden puuttuessa tämä arvo voidaan määrittää helposti epäsuorasti, kun tiedetään veriplasman natriumin, glukoosin ja urean pitoisuudet mmolina. Tämän kaavan käyttö on optimaalinen alkuhyperglykemian ja uremian yhteydessä. . Plasman osmolaarisuus (mOsm/L) =Na(mol/l) 1,86 + glukoosi (mmol/l)+ urea (mmol/l) + 10




Fysikaaliset ja kemialliset peruskäsitteet:

    Osmolaarisuus– aineen pitoisuusyksikkö, joka kuvastaa sen pitoisuutta yhdessä litrassa liuotinta.

    Osmolaliteetti– aineen pitoisuusyksikkö, joka kuvastaa sen pitoisuutta yhdessä kilogrammassa liuotinta.

    Vastaavuus– indikaattori, jota käytetään kliinisessä käytännössä kuvaamaan aineiden pitoisuutta dissosioituneessa muodossa. Yhtä kuin millimoolien lukumäärä kerrottuna valenssilla.

    Osmoottinen paine- paine, joka on kohdistettava veden liikkeen pysäyttämiseksi puoliläpäisevän kalvon läpi pitoisuusgradienttia pitkin.

Aikuisen ihmisen kehossa vesi muodostaa 60 % kehon painosta ja jakautuu kolmella pääsektorilla: solunsisäinen, solunulkoinen ja solujen välinen (suolen lima, seroosineste, aivo-selkäydinneste). Solunulkoinen tila sisältää intravaskulaariset ja interstitiaaliset osat. Solunulkoisen tilan kapasiteetti on 20 % kehon painosta.

Vesisektorien tilavuuksien säätö tapahtuu osmoosilakien mukaan, missä päärooli on natriumionilla, ja myös urean ja glukoosin pitoisuus on tärkeä. Normaali veriplasman osmolaarisuus on 282 – 295 mOsm/ l. Se lasketaan kaavan mukaan:

P osm = 2 Na + +2 TO + + Glukoosi + urea

Yllä oleva kaava heijastaa ns laskettu osmolaarisuus, jota säädellään lueteltujen komponenttien pitoisuudella ja liuottimena olevan veden määrällä.

Termi mitattu osmolaarisuus kuvastaa osmometrilaitteen määrittämää todellista arvoa. Jos siis mitattu osmolaarisuus ylittää lasketun, veriplasmassa kiertää huomioimattomia osmoottisesti aktiivisia aineita, kuten dekstraania, etyylialkoholia, metanolia jne..

Pääioni solunulkoisessa nesteessä on natrium. Normaali pitoisuus plasmassa 135-145 mmol/l. 70 % elimistön natriumin kokonaismäärästä on intensiivisesti mukana aineenvaihduntaprosesseissa ja 30 % sitoutuu luukudokseen. Useimmat solukalvot ovat natriumia läpäisemättömiä. Sen gradienttia ylläpidetään aktiivisella poistamisella soluista Na/K ATPaasin kautta

Munuaisissa 70 % natriumin kokonaismäärästä imeytyy takaisin proksimaaliseen tubulukseen ja vielä 5 % voi imeytyä takaisin distaaliseen tubulukseen aldosteronin vaikutuksesta.

Normaalisti kehoon tulevan nesteen tilavuus on yhtä suuri kuin siitä vapautuvan nesteen määrä. Päivittäinen nesteenvaihto on 2 - 2,5 litraa (taulukko 1).

Taulukko 1. Likimääräinen päivittäinen nestetasapaino

Sisäänpääsy

Valinta

polku

Määrä (ml)

polku

Määrä (ml)

Nesteiden ottaminen

Hiki

Aineenvaihdunta

Kaikki yhteensä

2000 - 2500

Kaikki yhteensä

2000 - 2500

Vesihävikki lisääntyy merkittävästi hypertermian (10 ml/kg kutakin yli 37 0 C astetta kohti), takypnean (10 ml/kg RR  20) ja mekaanisen hengityksen aikana ilman kostutusta.

DYSYDRIA

Veden aineenvaihduntahäiriöiden patofysiologia.

Rikkomukset voivat liittyä nesteen puutteeseen (dehydraatio) tai sen ylimäärään (ylihydraatio). Jokainen yllä olevista häiriöistä voi puolestaan ​​olla isotoninen (normaalilla plasman osmolaarisella), hypotoninen (kun plasman osmolaarisuus vähenee) ja hypertoninen (plasman osmolaarisuus ylittää merkittävästi sallitut normaalirajat).

Isotoninen nestehukka - on sekä veden puutetta että suolan puutetta. Plasman osmolaarisuus on normaali (270-295 mOsm/L). Solunulkoinen tila kärsii, sitä vähentää hypovolemia. Sitä havaitaan potilailla, joilla on menetyksiä maha-suolikanavasta (oksentelu, ripuli, fistelit), verenhukka, peritoniitti ja palovamma, polyuria, diureettien hallitsemattoman käytön yhteydessä.

Hypertensiivinen dehydraatio on tila, jolle on ominaista absoluuttinen tai vallitseva nesteen puute ja lisääntynyt plasman osmolaarisuus. Na > 150 mmol/l, plasman osmolaarisuus > 290 mOsm/l. Sitä havaitaan riittämättömällä veden saannilla (riittämätön putkiruokinta - 100 ml vettä tulee antaa jokaista 100 kcal:ta kohti), maha-suolikanavan sairaudet, hypotonisen nesteen menetys - keuhkokuume, trakeobronkiitti, kuume, trakeostomia, polyuria, osmodiureesi diabetes insipiduksessa.

Hypotoninen nestehukka - on veden puute ja hallitseva elektrolyyttihäviö. Solunulkoinen tila vähenee ja solut ylikyllästyvät vedellä. Na<13О ммоль/л, осмолярность плазмы < 275мосм/л. Наблюдается при состояниях, связанных с потерей солей (болезнь Аддисона, применение диуретиков, слабительных, осмодиурез, диета, бедная натрием), при введении избыточного количества инфузионных растворов, не содержащих электролиты (глюкоза, коллоиды).

Vesipula. Vesipula voi johtua joko riittämättömästä tarjonnasta tai liiallisista häviöistä. Saannin puute on kliinisessä käytännössä melko harvinaista.

Syitä vesihäviöiden lisääntymiseen:

1. Diabetes insipidus

Keski

Nefrogeeninen

2. Liiallinen hikoilu

3. Runsas ripuli

4. Hyperventilaatio

Tässä tapauksessa ei menetetä puhdasta vettä, vaan hypotonista nestettä. Solunulkoisen nesteen osmolaarisuuden lisääntyminen aiheuttaa solunsisäisen veden siirtymisen verisuoniin, mutta tämä ei täysin kompensoi hyperosmolaarisuutta, joka lisää antidiureettisen hormonin (ADH) pitoisuutta. Koska tällainen nestehukka kompensoituu osittain solunsisäisestä sektorista, kliiniset oireet ovat lieviä. Jos syy ei ole munuaisten menetys, virtsa tiivistyy.

Keskitason diabetes insipidus esiintyy usein neurokirurgian ja TBI:n jälkeen. Syynä on aivolisäkkeen tai hypotalamuksen vaurio, joka ilmenee ADH:n synteesin vähenemisenä. Taudille on ominaista polydipsia ja polyuria ilman glugosuriaa. Virtsan osmolaarisuus on pienempi kuin plasman osmolaarisuus.

Nefrogeeninen diabetes insipidus kehittyy useimmiten kroonisen munuaissairauden seurauksena ja joskus nefrotoksisten lääkkeiden (amfoterisiini B, litium, demeklosykliini, mannitoli) sivuvaikutuksena. Syynä on munuaisten tubulusreseptorien herkkyyden väheneminen vasopressiinille. Sairauden kliiniset ilmenemismuodot ovat samat, ja diagnoosi vahvistetaan sillä, että diureesin nopeus ei vähene ADH:ta annettaessa.

Natriumin puute.

Natriumin puutteen syynä voi olla joko liiallinen erittyminen tai riittämätön saanti. Erittyminen puolestaan ​​voi tapahtua munuaisten, suoliston ja ihon kautta.

Natriumin puutteen syyt:

1. Menetys munuaisten kautta

Akuutin munuaisten vajaatoiminnan polyuriavaihe;

Diureettien käyttö

Mineralokortikoidien puutos

Osmodiureesi (esimerkiksi diabetes mellituksessa)

2. Menetys ihon läpi

ihotulehdus;

Kystinen fibroosi.

3. Tappio suoliston kautta

Suolen ahtauma, peritoniitti.

4. Runsaasti suoloja sisältävän nesteen menetys, joka kompensoituu suolattomilla liuoksilla (runsas ripuli kompensoituu 5-prosenttisella glukoosiliuoksella).

Natrium voi kadota hypo- tai isotonisiin nesteisiin. Molemmissa tapauksissa solunulkoisen tilan tilavuus vähenee, mikä johtaa tilavuusreseptorien ärsytykseen ja aldosteronin vapautumiseen. Lisääntynyt natriumin retentio aiheuttaa protonien erittymisen lisääntymistä nefronitiehyen onteloon ja bikarbonaatti-ionien reabsorptiota (katso happo-emäs-säätelyn munuaismekanismit), ts. aiheuttaa metabolista alkaloosia.

Kun natriumia häviää, sen pitoisuus plasmassa ei heijasta kehon kokonaispitoisuutta, koska se riippuu siihen liittyvästä veden menetyksestä. Joten jos se katoaa hypotoniseen nesteeseen, plasman pitoisuus on normaalia korkeampi; jos se häviää yhdessä vedenpidätyksen kanssa, se on pienempi. Saman määrän natriumin ja veden menetys ei vaikuta sen plasmatasoihin. Taulukossa 2 on esitetty diagnoosi vesi- ja natriumhäviöiden vallitsemisesta.

Taulukko 2. Vallitsevien vesi- tai natriumhäviöiden diagnoosi

Jos vesihäviö on vallitsevaa, solunulkoisen nesteen osmolaarisuus kasvaa, mikä aiheuttaa veden siirtymisen soluista interstitiumiin ja suoniin. Siksi kliiniset oireet ilmaistaan ​​vähemmän selvästi.

Tyypillisin tapaus on natriumin menetys isotonisessa nesteessä (isotoninen dehydraatio). Solunulkoisen sektorin kuivumisasteesta riippuen kliinisessä kuvassa erotetaan kolme dehydraatioastetta (taulukko 3).

Taulukko 3: Kuivumisasteen kliininen diagnoosi.

Ylimääräinen vesi.

Ylimääräinen vesi liittyy erittymisen heikkenemiseen, ts. munuaisten vajaatoiminta. Terveiden munuaisten kyky erittää vettä on 20 ml/tunti, joten jos niiden toiminta ei ole heikentynyt, ylimääräinen vesi on käytännössä poissuljettua. Vesimyrkytyksen kliiniset merkit johtuvat ensisijaisesti aivoturvotuksesta. Sen esiintymisvaara syntyy, kun natriumpitoisuus lähestyy 120 mmol/l.

Elektrolyytit ovat ihmiskehossa olevia ioneja, jotka sisältävät sähkövarauksia. Ihmiskehon neljä tunnetuinta elektrolyyttiä ovat natrium, kalium, kalsium ja magnesium. Niillä on keskeinen rooli kehon normaalin toiminnan varmistamisessa. Jos epäilet, että saatat kärsiä elektrolyyttitasapainon häiriöstä, lue tämä artikkeli saadaksesi lisätietoja tämän häiriön oireista ja sen hoidosta.

Askeleet

Arvioi elektrolyyttitasot

Yleisimmät elektrolyytit ovat natrium, kalium, kalsium ja magnesium. Kun näiden elektrolyyttien tasot kehossasi muuttuvat epätasapainoksi, sitä kutsutaan elektrolyyttiepätasapainoksi.

    Huomaa natriumin puutteen oireet kehossasi. Natrium on yksi ihmiskehon runsaimmista elektrolyyteistä. Kun elektrolyyttitasot ovat tasapainossa, veresi sisältää 135-145 mmol/l natriumia. Saat eniten natriumia suolaisista ruoista. Siksi, kun kehosi natriumtasot ovat alhaiset (kutsutaan hyponatremiaksi), kaipaat suolaisia ​​ruokia.

    • Oireet: Kaipaat suolaisia ​​ruokia. Muita hyponatremian oireita ovat erittäin väsynyt tunne, lihasheikkous ja lisääntynyt virtsaaminen.
    • Kun kehosi natriumtasot laskevat liian alhaiseksi, saatat saada sydänkohtauksen, et pysty hengittämään ja jopa joudut koomaan. Näitä oireita esiintyy kuitenkin vain äärimmäisissä tilanteissa.
  1. Ole tietoinen ylimääräisen natriumin oireista kehossasi. Kuten jo mainittiin, veren normaali natriumpitoisuus on 135-145 mmol/l. Kun natriumin määrä ylittää 145 mmol/l, sitä kutsutaan hypernatremiaksi. Nesteen menetys oksentamisen, ripulin ja palovammojen seurauksena voi johtaa tähän tilaan. Voit myös saada liikaa natriumia, jos et juo tarpeeksi vettä tai syö liikaa suolaista ruokaa.

    • Oireet: Sinulla on jano ja suusi on hyvin kuiva. Saatat huomata, että lihaksesi alkavat nykiä, tuntea ärtyneisyyttä ja saattaa olla hengitysvaikeuksia.
    • Äärimmäisen ylimääräisellä natriumilla voi esiintyä kouristuksia ja tajunnan tasoa.
  2. Varo kaliumin puutetta. 98 % kehon kaliumista löytyy solujen sisältä, ja veressäsi on 3,5-5 mmol/l kaliumia. Kalium edistää tervettä luuston ja lihasten liikettä sekä sydämen normaalia toimintaa. Hypokalemia tarkoittaa alhaista kaliumtasoa elimistössä (alle 3,5 mmol/l). Tämä voi tapahtua, kun hikoilet liikaa harjoituksen aikana tai jos käytät laksatiiveja.

    • Oireet: Tunnet olevasi väsynyt ja heikko. Saatat myös kokea ummetusta, jalkakramppeja ja heikentynyttä jännerefleksiä.
    • Jos sinulla on erittäin vähän kaliumia, saatat kokea epäsäännöllistä sydämenlyöntiä, joka tunnetaan myös nimellä rytmihäiriö.
  3. Kiinnitä huomiota lihasheikkouteen, sillä se voi olla merkki liiallisesta kaliumista. Tyypillisesti ylimääräinen kalium voi johtua vain sairaudesta, kuten munuaisten vajaatoiminnasta ja diabeteksesta.

    • Oireet: Tunnet olosi erittäin heikoksi, koska liiallinen kalium johtaa lihasheikkouteen. Saatat myös kokea pistelyä ja tunnottomuutta lihaksissasi. Joissakin tapauksissa saatat myös kokea hämmennystä.
    • Erittäin liiallinen kaliumtaso voi aiheuttaa epäsäännöllisiä sydämenlyöntejä, jotka vaikeimmissa tapauksissa voivat johtaa sydänkohtaukseen.
  4. Kiinnitä huomiota kalsiumin puutteen merkkeihin. Kalsium saattaa olla tunnetuin elektrolyytti. Sitä löytyy useimmista maitotuotteista ja se vahvistaa luita ja hampaita. Normaali kalsiumtaso veressä on 2,25-2,5 mmol/l. Kun kalsiumtaso laskee tämän tason alapuolelle, kehittyy hypokalsemia.

    • Oireet: Hypokalsemia voi aiheuttaa lihaskramppeja ja vapinaa. Luusi voivat muuttua hauraiksi ja heikoksi.
    • Saatat kokea epäsäännöllisiä sydämenlyöntejä tai kouristuskohtauksia, jos kehosi kalsiumtasot ovat liian alhaiset pitkään aikaan.
  5. Tarkkaile kehosi ylimääräisen kalsiumin oireita. Kun veren kalsiumpitoisuus ylittää 2,5 mmol/l, sitä kutsutaan hyperkalsemiaksi. Lisäkilpirauhashormoni (PTH) on vastuussa kalsiumin tuotannosta kehossa. Kun lisäkilpirauhashormoni muuttuu liian aktiiviseksi (hyperparatyreoosissa), kehoon muodostuu ylimääräistä kalsiumia. Tämä voi johtua myös pitkistä immobilisaatiojaksoista.

    • Oireet: Lievä hyperkalsemia (pieni ylimäärä kalsiumia veressä) ei yleensä aiheuta oireita. Jos kalsiumtasosi nousee kuitenkin edelleen, saatat kokea heikkoutta, luukipua ja ummetusta.
    • Vaikeissa tapauksissa sinulle voi kehittyä munuaiskiviä, jos jätät hyperkalsemiaa hoitamatta.
  6. Tarkkaile alhaista magnesiumtasoa sairaalassa ollessasi. Magnesium on neljänneksi runsain elektrolyytti kehossasi. Keskimääräinen magnesiumpitoisuus ihmiskehossa on 24 g, ja 53 % tästä määrästä löytyy luista. Hypomagnesemiaa havaitaan yleensä potilailla, jotka ovat olleet sairaalahoidossa, ja hyvin harvoin potilailla, jotka eivät ole sairaalahoidossa.

    • Oireet: Oireita ovat lievä vapina, sekavuus ja nielemisvaikeudet.
    • Vakavia oireita ovat hengitysvaikeudet, anoreksia ja kouristukset.
  7. Tiedä, että ylimääräinen magnesium on harvinaista myös ei-sairaalahoidossa olevilla ihmisillä. Hypermagnesemia on tila, jossa ihmiskehoon muodostuu ylimääräistä magnesiumia. Tämä on hyvin harvinainen tila, ja sitä esiintyy yleensä vain potilailla, jotka ovat sairaalahoidossa. Kuivuminen, luusyöpä, hormonaalinen epätasapaino ja munuaisten vajaatoiminta ovat yleisimpiä hypermagnesemian syitä.

    • Oireet: Ihosi voi muuttua punaiseksi ja kosketettavaksi lämpimäksi. Saatat myös kokea refleksien heikkenemistä, heikkoutta ja oksentelua.
    • Vakavia oireita ovat kooma, halvaus ja hypoventilaatio-oireyhtymä. On myös mahdollista, että sykkeesi saattaa hidastua.

    Elektrolyyttitasapainon hoito

    1. Nosta natriumtasojasi. Ensinnäkin: lepää, normalisoi hengityksesi ja rentoudu. Todennäköisesti sinun täytyy vain syödä jotain suolaista, joten istu alas ja syö. Lievät natriumin puutteen oireet alkavat yleensä siitä, että et ole syönyt mitään suolaista vähään aikaan. Voit myös juoda elektrolyyteillä väkevöityä juomaa.

      Laske natriumtasojasi. Istu alas ja juo lasillinen vettä. Suurin osa ylimääräiseen natriumiin liittyvistä oireista johtuu liiallisesta suolaisen ruoan syömisestä. Juo runsaasti vettä, kunnes olet täysin janoton. Oksentelu voi myös johtaa kuivumiseen, joten jos tunnet pahoinvointia, hoida pahoinvoinnin syy ja ole varovainen syömisissäsi.

      • Jos alat kouristella, soita ambulanssi.
    2. Nosta kaliumtasojasi. Jos kaliumpuutosi johtuu liiallisesta hikoilusta tai oksentamisesta, juo runsaasti nesteitä kehosi nesteytyksestä. Jos koet hypokalemian oireita harjoituksen aikana, pysähdy, istu alas ja juo elektrolyyttipitoista juomaa. Jos tunnet lihaskouristuksen, venytä sitä. Voit myös palauttaa normaalit kaliumtasot veressäsi syömällä runsaasti kaliumia sisältäviä ruokia.

      Alentaa kehosi magnesiumtasoja. Jos sinulla on vain lieviä hypermagnesemian oireita, juo runsaasti vettä ja lopeta magnesiumpitoisten ruokien syöminen muutamaksi päiväksi. Korkeat magnesiumpitoisuudet havaitaan kuitenkin useimmiten munuaissairauden oireena. Sinun on hoidettava taustalla oleva sairaus normalisoidaksesi kehosi magnesiumtasot. Keskustele lääkärisi kanssa parhaan hoitovaihtoehdon määrittämiseksi.

      • Jos sinulla on ollut sydänsairaus ja sydämen syke on epäsäännöllinen, hakeudu välittömästi lääkärin hoitoon.
    3. Vahvista luita nostamalla kalsiumtasoa. Lieviä tai kohtalaisia ​​kalsiumin puutoksen oireita voidaan yleensä lievittää syömällä kalsiumilla täydennettyjä ruokia. Voit myös lisätä D-vitamiinin saantia, joka parantaa kehosi kalsiumin käyttöä viettämällä 30 minuuttia auringossa ennen klo 8.00. Auringossa oleskelu klo 8 jälkeen voi johtaa tiettyihin terveysongelmiin. Voit myös ottaa D-vitamiinia ravintolisänä. Jos tunnet lihaskouristuksia, venytä ja hiero niitä.

      Vähennä kalsiumin määrää kehossasi. Jos sinulla on vain lieviä liiallisen kalsiumin oireita, juo tarpeeksi vettä ja syö runsaasti kuitua sisältäviä ruokia ummetuksen lievittämiseksi. Sinun tulee välttää runsaasti kalsiumia sisältävien elintarvikkeiden syömistä. Liiallinen kalsium johtuu yleensä hyperparatyreoosista, josta sinun on päästävä eroon ennen kuin voit alentaa kehosi kalsiumtasoja. Keskustele lääkärisi kanssa hoitovaihtoehdoista.

KIRURGISET POTILAATITJA INFUUSIOHOIDON PERIAATTEET

Akuutit vesi- ja elektrolyyttitasapainon häiriöt ovat yksi yleisimmistä kirurgisen patologian komplikaatioista - vatsakalvontulehdus, suolistotukos, haimatulehdus, trauma, sokki, sairaudet, joihin liittyy kuumetta, oksentelua ja ripulia.

9.1. Tärkeimmät syyt vesi- ja elektrolyyttitasapainohäiriöihin

Tärkeimmät rikkomusten syyt ovat:

    ulkoiset neste- ja elektrolyyttihäviöt ja niiden patologinen uudelleenjakautuminen tärkeimpien nesteympäristöjen välillä kehon luonnollisten prosessien patologisen aktivoitumisen vuoksi - polyuria, ripuli, liiallinen hikoilu, runsas oksentelu, erilaisten viemärien ja fistelien kautta tai haavojen pinnasta ja palovammoja;

    nesteiden sisäinen liike vaurioituneiden ja infektoituneiden kudosten turvotuksen aikana (murtumat, puristusoireyhtymä); nesteen kertyminen keuhkopussin (keuhkopussintulehdus) ja vatsan (peritoniitti) onteloihin;

    nesteiden osmolaarisuuden muutokset ja ylimääräisen veden liikkuminen soluun tai sieltä ulos.

nesteen liikkuminen ja kertyminen maha-suolikanavaan, usean litran saavuttaminen (suolitukoksen, suolistoinfarktin sekä vaikean postoperatiivisen pareesin kanssa) vastaa vakavuudeltaan patologista prosessia ulkoiset tappiot nesteitä, koska molemmissa tapauksissa menetetään suuria määriä nestettä, jossa on korkea elektrolyytti- ja proteiinipitoisuus. Yhtä merkittävät plasman kanssa identtisen nesteen ulkoiset häviöt haavojen ja palovammojen pinnalta (lantiononteloon) sekä laajojen gynekologisten, proktologisten ja rintakehän leikkausten aikana (keuhkopussin onteloon).

Sisäinen ja ulkoinen nestehäviö määrittelee kliinisen kuvan nesteen puutteesta ja vesi-elektrolyyttiepätasapainosta: hemokonsentraatio, plasman puutos, proteiinin menetys ja yleinen kuivuminen. Kaikissa tapauksissa nämä häiriöt vaativat kohdennettua vesi- ja elektrolyyttitasapainon korjaamista. Tuntemattomina ja ratkaisemattomina ne huonontavat potilaiden hoidon tuloksia.

Koko kehon vesivarasto sijaitsee kahdessa tilassa - solunsisäisessä (30-40% ruumiinpainosta) ja solunulkoisessa (20-27% ruumiinpainosta).

Solunulkoinen volyymi jakautuu interstitiaaliseen veteen (nivelsiteiden, ruston, luiden, sidekudoksen, imusolmukkeiden, plasman vesi) ja aineenvaihduntaprosesseihin aktiivisesti osallistumattoman veden välillä (aivo-selkäydinneste, nivelneste, maha-suolikanavan sisältö).

Solunsisäinen sektori sisältää vettä kolmessa muodossa (konstitutiivinen, protoplasma ja kolloidiset misellit) ja siihen liuenneita elektrolyyttejä. Soluvesi jakautuu epätasaisesti eri kudoksiin, ja mitä hydrofiilisempiä ne ovat, sitä herkempiä ne ovat veden aineenvaihdunnan häiriöille. Osa soluvedestä muodostuu aineenvaihduntaprosessien seurauksena.

Päivittäinen aineenvaihduntaveden tilavuus 100 g proteiinien, rasvojen ja hiilihydraattien "polton" aikana on 200-300 ml.

Solunulkoisen nesteen tilavuus voi kasvaa loukkaantumisen, paaston, sepsiksen, vakavien infektiosairauksien, eli sellaisten tilojen, joihin liittyy merkittävä lihasmassan menetys, yhteydessä. Solunulkoisen nesteen tilavuus lisääntyy turvotuksen aikana (sydän, proteiiniton, tulehduksellinen, munuainen jne.).

Solunulkoisen nesteen tilavuus pienenee kaikenlaisen kuivumisen yhteydessä, erityisesti suolojen häviämisen yhteydessä. Merkittäviä häiriöitä havaitaan kirurgisilla potilailla kriittisissä olosuhteissa - vatsakalvontulehdus, haimatulehdus, verenvuotoshokki, suolitukos, verenhukka, vakava trauma. Tällaisten potilaiden vesi- ja elektrolyyttitasapainon säätelyn perimmäinen tavoite on ylläpitää ja normalisoida verisuonten ja interstitiaalien tilavuutta, niiden elektrolyytti- ja proteiinikoostumusta.

Solunulkoisen nesteen tilavuuden ja koostumuksen ylläpito ja normalisointi ovat valtimo- ja keskuslaskimopaineen, sydämen minuuttitilavuuden, elinten verenkierron, mikroverenkierron ja biokemiallisen homeostaasin säätelyn perusta.

Elimistön vesitasapainon ylläpitäminen tapahtuu normaalisti riittävällä veden saannilla sen hävikkien mukaisesti; Päivittäinen "liikevaihto" on noin 6 % kehon kokonaisvedestä. Aikuinen kuluttaa noin 2500 ml vettä päivässä, josta 300 ml aineenvaihduntaprosessien tuloksena syntyvää vettä. Vedenhukkaa on noin 2500 ml/vrk, josta 1500 ml erittyy virtsaan, 800 ml haihtuu (400 ml hengitysteiden kautta ja 400 ml ihon kautta), 100 ml hikeen ja 100 ml ulosteisiin. Korjaavaa infuusio-siirtohoitoa ja parenteraalista ravitsemusta suoritettaessa mekanismit, jotka säätelevät nesteen virtausta ja kulutusta sekä janoa, ovat ohimeneviä. Siksi kliinisten ja laboratoriotietojen, ruumiinpainon ja päivittäisen virtsanerityksen huolellinen seuranta on tarpeen normaalin nesteytystilan palauttamiseksi ja ylläpitämiseksi. On huomattava, että fysiologiset vaihtelut vesihäviössä voivat olla varsin merkittäviä. Kun kehon lämpötila nousee, endogeenisen veden määrä kasvaa ja veden menetys ihon läpi hengityksen aikana lisääntyy. Hengityshäiriöt, erityisesti hyperventilaatio alhaisella ilmankosteudella, lisäävät elimistön vedentarvetta 500-1000 ml. Nestehävikki laajoista haavapinnoista tai pitkittyneiden vatsa- ja rintaonteloiden kirurgisten toimenpiteiden aikana yli 3 tunnin ajan lisää veden tarpeen 2500 ml:aan/vrk.

Jos veden saanti ylittää sen vapautumisen, lasketaan vesitase positiivinen; erityselinten toiminnallisten häiriöiden taustalla siihen liittyy turvotuksen kehittyminen.

Kun veden vapautuminen on enemmän kuin saanti, saldo lasketaan negatiivinen- tässä tapauksessa janon tunne toimii signaalina kuivumisesta.

Kuivumisen ennenaikainen korjaus voi johtaa romahdukseen tai dehydraatioshokkiin.

Tärkein vesi- ja elektrolyyttitasapainoa säätelevä elin on munuaiset. Erittyvän virtsan määrä määräytyy elimistöstä poistettavien aineiden määrän ja munuaisten kyvyn tiivistää virtsaa.

300-1500 mmol aineenvaihdunnan lopputuotteita erittyy virtsaan päivässä. Veden ja elektrolyyttien puutteessa oliguria ja anuria häviävät

katsotaan fysiologisena vasteena, joka liittyy ADH:n ja aldosteronin stimulaatioon. Vesi- ja elektrolyyttihäviöiden korjaaminen johtaa diureesin palautumiseen.

Normaalisti vesitasapainon säätely tapahtuu aktivoimalla tai estämällä hypotalamuksen osmoreseptoreita, jotka reagoivat plasman osmolaarisuuden muutoksiin, janon tunteen ilmaantuvuuteen tai heikkenemiseen sekä aivolisäkkeen antidiureettisen hormonin (ADH) erittymiseen. muuttuu vastaavasti. ADH lisää veden takaisinimeytymistä munuaisten distaalisissa tubuluksissa ja keräyskanavissa ja vähentää virtsan eritystä. Päinvastoin, ADH-erityksen vähenemisen myötä virtsaaminen lisääntyy ja virtsan osmolaarisuus vähenee. ADH:n muodostuminen lisääntyy luonnollisesti nestetilavuuden pienentyessä interstitiaalisilla ja intravaskulaarisilla sektoreilla. Veren tilavuuden lisääntyessä ADH:n eritys vähenee.

Patologisissa tiloissa tärkeitä ovat esimerkiksi hypovolemia, kipu, traumaattiset kudosvauriot, oksentelu ja lääkkeet, jotka vaikuttavat vesi- ja elektrolyyttitasapainon hermoston säätelyn keskusmekanismeihin.

Kehon eri osien nestemäärän ja tilan välillä on läheinen suhde perifeerinen verenkierto, kapillaariläpäisevyys ja kolloidi-osmoottisen ja hydrostaattisen paineen suhde.

Normaalisti nesteen vaihto verisuonikerroksen ja interstitiaalisen tilan välillä on tiukasti tasapainossa. Patologisissa prosesseissa, jotka liittyvät ensisijaisesti plasmassa kiertävän proteiinin häviämiseen ( akuutti verenhukka, maksan vajaatoiminta), plasman CODE laskee, minkä seurauksena ylimääräinen neste mikroverenkiertojärjestelmästä siirtyy interstitiumiin. Veri paksuuntuu ja sen reologiset ominaisuudet häiriintyvät.

9.2. Elektrolyyttiaineenvaihdunta

Veden aineenvaihdunnan tila normaaleissa ja patologisissa olosuhteissa liittyy läheisesti elektrolyyttien - Na +, K +, Ca 2+, Mg 2+, SG, HC0 3, H 2 P0 4 ~, SOf sekä proteiinien - vaihtoon. ja orgaaniset hapot.

Elektrolyyttien pitoisuus kehon nestetiloissa ei ole sama; plasma ja interstitiaalinen neste eroavat merkittävästi vain proteiinipitoisuudessa.

Elektrolyyttipitoisuus solunulkoisessa ja intrasellulaarisessa nestetilassa ei ole sama: solunulkoinen sisältää pääasiassa Na +, SG, HCO^; solunsisäisessä - K+, Mg+ ja H2PO4; S0 4 2 :n ja proteiinien pitoisuus on myös korkea. Erot tiettyjen elektrolyyttien pitoisuuksissa muodostavat lepobiosähköisen potentiaalin, joka antaa hermo-, lihas- ja sektorisoluille kiihtyvyyttä.

Sähkökemiallisen potentiaalin säilyttäminen solu- ja ekstrasellulaarinentilaa varmistetaan Na + -, K + -ATPaasipumpun toiminnalla, jonka ansiosta Na + "pumppataan" jatkuvasti ulos solusta ja K + - "ajetaan" siihen niiden pitoisuusgradientteja vastaan.

Kun tämä pumppu häiriintyy hapen puutteen tai aineenvaihduntahäiriöiden seurauksena, solutila vapautuu natriumille ja kloorille. Siihen liittyvä osmoottisen paineen nousu solussa lisää veden liikettä solussa aiheuttaen turvotusta,

ja sen jälkeen kalvon eheyden rikkominen lyysiin asti. Siten hallitseva kationi solujen välisessä tilassa on natrium ja solussa - kalium.

9.2.1. Natriumaineenvaihdunta

Natrium - tärkein ekstrasellulaarinen kationi; interstitiaalisen tilan tärkein kationi on tärkein osmoottisesti aktiivinen aine plasmassa; osallistuu toimintapotentiaalien syntymiseen, vaikuttaa solunulkoisten ja solunsisäisten tilojen tilavuuteen.

Kun Na + -pitoisuus pienenee, osmoottinen paine laskee samalla kun interstitiaalisen tilan tilavuus pienenee. Natriumpitoisuuden nousu aiheuttaa päinvastaisen prosessin. Natriumin puutetta ei voida kompensoida millään muulla kationilla. Aikuisen natriumin päivittäinen tarve on 5-10 g.

Natrium erittyy elimistöstä pääasiassa munuaisten kautta; pieni osa tulee hikoilusta. Sen taso veressä kohoaa pitkäaikaisessa kortikosteroidihoidossa, pitkäaikaisessa mekaanisessa ventilaatiossa hyperventilaatiotilassa, diabetes insipiduksessa ja hyperaldosteronismissa; vähenee diureettien pitkäaikaisen käytön vuoksi, pitkäaikaisen hepariinihoidon taustalla, kroonisen sydämen vajaatoiminnan, hyperglykemian ja maksakirroosin yhteydessä. Virtsan normaali natriumpitoisuus on 60 mmol/l. Antidiureettisten mekanismien aktivoitumiseen liittyvä kirurginen aggressio johtaa natriumin retentioon munuaisten tasolla, joten sen pitoisuus virtsassa voi laskea.

Hypernatremia(plasmanatrium yli 147 mmol/l) esiintyy kohonneella natriumpitoisuudella interstitiaalisessa tilassa, mikä johtuu nestehukasta, elimistön suolan liikakuormituksesta ja diabetes insipiduksesta. Hypernatremiaan liittyy nesteen uudelleenjakautuminen solunsisäisestä solunulkoiseen sektoriin, mikä aiheuttaa solujen kuivumista. Kliinisessä käytännössä tämä tila ilmenee lisääntyneen hikoilun, hypertonisen natriumkloridiliuoksen suonensisäisen infuusion ja myös akuutin munuaisten vajaatoiminnan kehittymisen vuoksi.

Hyponatremia(plasmanatrium alle 136 mmol/l) kehittyy liiallisella ADH:n erittymisellä vasteena kiputekijälle, patologiseen nesteen menetykseen maha-suolikanavan kautta, suolattomien tai glukoosiliuosten liiallisella suonensisäisellä annosta, liiallinen vedenotto taustalla rajoitettu ruoan saanti; johon liittyy solujen hyperhydraatio ja samanaikainen BCC:n väheneminen.

Natriumin puute määritetään kaavalla:

Alijäämälle (mmol) = (Na HOpMa - todellinen luku) ruumiinpaino (kg) 0,2.

9.2.2. Kaliumin aineenvaihdunta

kalium - tärkein intrasellulaarinen kationi. Päivittäinen kaliumtarve on 2,3-3,1 g Kalium (yhdessä natriumin kanssa) osallistuu aktiivisesti kaikkiin aineenvaihduntaprosesseihin kehon prosesseja. Kaliumilla, kuten natriumilla, on johtava rooli kalvopotentiaalien muodostumisessa; se vaikuttaa pH-arvoon ja glukoosin käyttöön ja on välttämätön proteiinisynteesille.

Leikkauksen jälkeisellä kaudella kriittisissä olosuhteissa kaliumhäviöt voivat ylittää sen saannin; ne ovat tyypillisiä myös pitkäaikaiselle paastolle, johon liittyy kehon solumassan menetys - kaliumin tärkein "varasto". Maksan glykogeeniaineenvaihdunnalla on tietty rooli kaliumhäviön lisäämisessä. Vakavasti sairailla potilailla (ilman asianmukaista korvausta) jopa 300 mmol kaliumia siirtyy solutilasta solunulkoiseen tilaan viikossa. Varhaisessa posttraumaattisessa jaksossa kalium poistuu solusta metabolisen typen mukana, jonka ylimäärä muodostuu solun proteiinikatabolismin seurauksena (keskimäärin 1 g typpeä "kantaa" 5-6 meq kaliumia.

minämunkki.temia(plasman kalium alle 3,8 mmol/l) voi kehittyä natriumylimäärällä, metabolisen alkaloosin taustalla, hypoksialla, vakavalla proteiinikatabolialla, ripulilla, pitkittyneellä oksentelulla jne. Solunsisäisen kaliumin puutteen yhteydessä Na + ja H + solua intensiivisesti, mikä aiheuttaa solunsisäistä asidoosia ja hyperhydraatiota solunulkoisen metabolisen alkaloosin taustalla. Kliinisesti tämä tila ilmenee rytmihäiriönä, valtimoverenpaineena, luuston lihasten tonuksen heikkenemisenä, suoliston pareesina ja mielenterveyshäiriöinä. EKG:ssä näkyy tyypillisiä muutoksia: takykardia, kompleksin kapeneminen QRS, hampaan litistäminen ja kääntäminen T, hampaiden amplitudin kasvu U. Hypokalemian hoito alkaa poistamalla etiologinen tekijä ja kompensoimalla kaliumin puute kaavalla:

Kaliumin puute (mmol/l) = K + potilasplasma, mmol/l 0,2 paino, kg.

Suurien määrien kaliumvalmisteiden nopea antaminen voi aiheuttaa sydänkomplikaatioita, mukaan lukien sydämenpysähdyksen, joten vuorokausiannos ei saa ylittää 3 mmol/kg/vrk ja infuusionopeus 10 mmol/h.

Käytetyt kaliumvalmisteet tulee laimentaa (enintään 40 mmol 1 litraa injektoitua liuosta kohti); on optimaalista antaa ne polarisoivan seoksen muodossa (glukoosi + kalium + insuliini). Käsittely kaliumvalmisteilla suoritetaan päivittäisessä laboratoriovalvonnassa.

Hyperkalemia(plasman kalium yli 5,2 mmol/l) esiintyy useimmiten silloin, kun kaliumin erittyminen elimistöstä on häiriintynyt (akuutti munuaisten vajaatoiminta) tai kun sitä vapautuu massiivisesti vaurioituneista soluista laajan trauman, punasolujen hemolyysin vuoksi , palovammat, asentopuristusoireyhtymä jne. Lisäksi hyperkalemia on tyypillistä hypertermialle, kouristavalle oireyhtymälle ja siihen liittyy useiden lääkkeiden käyttöä - hepariinia, aminokapronihappoa jne.

Diagnostiikka hyperkalemia perustuu etiologisten tekijöiden (trauma, akuutti munuaisten vajaatoiminta) esiintymiseen, tyypillisten muutosten esiintymiseen sydämen toiminnassa: sinusbradykardia (sydämenpysähdykseen asti) yhdessä kammioiden ekstrasystolen kanssa, laskimonsisäisen ja atrioventrikulaarisen johtumisen selvä hidastuminen ja tyypillinen laboratorio tiedot (plasman kalium yli 5,5 mmol/l). Korkea, terävä aalto tallennetaan EKG:hen T, kompleksin laajentaminen QRS, hampaiden amplitudin pieneneminen R.

Hoito hyperkalemia alkaa etiologisen tekijän poistamisella ja asidoosin korjaamisella. Kalsiumlisät on määrätty; Ylimääräisen plasman kaliumin siirtämiseksi soluun ruiskutetaan suonensisäisesti glukoosiliuosta (10-15 %) insuliinilla (1 yksikkö jokaista 3-4 g glukoosia kohden). Jos nämä menetelmät eivät tuota toivottua vaikutusta, hemodialyysi on aiheellinen.

9.2.3. Kalsiumin aineenvaihdunta

Kalsium on suunnilleen 2 % ruumiinpainosta, josta 99 % on luissa sitoutuneessa tilassa eivätkä normaaliolosuhteissa osallistu elektrolyyttiaineenvaihduntaan. Kalsiumin ionisoitu muoto osallistuu aktiivisesti neuromuskulaariseen virityksen välitykseen, veren hyytymisprosesseihin, sydänlihaksen työhön, solukalvojen sähköpotentiaalin muodostukseen ja useiden entsyymien tuotantoon. Päivittäinen tarve on 700-800 mg. Kalsium pääsee elimistöön ruoan mukana, erittyy maha-suolikanavan kautta ja virtsaan. Kalsiumin aineenvaihdunta liittyy läheisesti fosforin aineenvaihduntaan, plasman proteiinitasoihin ja veren pH:hon.

Hypokalsemia(plasman kalsium alle 2,1 mmol/l) kehittyy hypoalbuminemiaan, haimatulehdukseen, suurten sitraattiveren siirtoon, pitkäaikaisiin sappifisteleihin, D-vitamiinin puutteeseen, imeytymishäiriöön ohutsuolessa, erittäin traumaattisten leikkausten jälkeen. Kliinisesti ilmenee lisääntyneenä hermo-lihashermoitumisena, parestesiana, kohtauksellisena takykardiana, tetaniana. Hypokalsemia korjataan sen jälkeen, kun sen taso veriplasmassa on määritetty laboratoriossa antamalla suonensisäisesti ionisoitua kalsiumia (glukonaattia, laktaattia, kloridia tai kalsiumkarbonaattia) sisältäviä lääkkeitä. Hypokalsemian korjaavan hoidon tehokkuus riippuu albumiinitasojen normalisoitumisesta.

Hyperkalsemia(plasman kalsium yli 2,6 mmol/l) esiintyy kaikissa prosesseissa, joihin liittyy lisääntynyt luun tuhoutuminen (kasvaimet, osteomyeliitti), lisäkilpirauhasen sairaudet (adenooma tai lisäkilpirauhastulehdus), liiallinen kalsiumlisän antaminen sitraattiverensiirron jälkeen jne. Kliininen tila ilmenee lisääntyneenä väsymyksenä, letargiana ja lihasheikkoutena. Kun hyperkalsemia lisääntyy, ilmaantuu maha-suolikanavan atonian oireita: pahoinvointia, oksentelua, ummetusta, ilmavaivat. EKG:ssä näkyy tyypillistä intervallin lyhenemistä (2-7; rytmi- ja johtumishäiriöt, sinusbradykardia, eteiskammioiden johtumisen hidastuminen ovat mahdollisia; G-aalto voi muuttua negatiiviseksi, kaksivaiheiseksi, vähentyneeksi, pyöristyneeksi.

Hoito on vaikuttaa patogeneettiseen tekijään. Vaikeassa hyperkalsemiassa (yli 3,75 mmol/l) tarvitaan kohdennettua korjausta - 2 g (EDTA) dinatriumsuolaa laimennettuna 500 ml:aan 5 % glukoosiliuosta, annetaan hitaasti laskimoon, tipoittain 2-4 kertaa päivässä verenplasman kalsiumpitoisuuden hallinnassa.

9.2.4. Magnesiumin aineenvaihdunta

Magnesium on solunsisäinen kationi; sen pitoisuus plasmassa on 2,15 kertaa pienempi kuin punasolujen sisällä. Mikroelementti vähentää hermo-lihashermostoa ja sydänlihaksen supistumiskykyä ja aiheuttaa keskushermoston lamaa. Magnesiumilla on valtava rooli solujen hapen imeytymisessä, energiantuotannossa jne. Se pääsee kehoon ruoan mukana ja erittyy ruoansulatuskanavan ja virtsan kautta.

Hypomagnesemia(plasman magnesium alle 0,8 mmol/l) havaitaan maksakirroosissa, kroonisessa alkoholismissa, akuutissa haimatulehduksessa, akuutin munuaisten vajaatoiminnan polyuriavaiheessa, suolistofisteleissä, epätasapainoisessa infuusiohoidossa. Kliinisesti hypomagnesemia ilmenee lisääntyneenä hermostuneisuutena

lihasten kiihtyvyys, hyperrefleksia, eri lihasryhmien kouristussupistukset; Spastista kipua maha-suolikanavassa, oksentelua ja ripulia voi esiintyä. Hoito koostuu kohdistetusta vaikutuksesta etiologiseen tekijään ja magnesiumsuolojen antamisesta laboratoriovalvonnassa.

Hypermagnesemia(plasman magnesium yli 1,2 mmol/l) kehittyy ketoasidoosin, lisääntyneen katabolian, akuutin munuaisten vajaatoiminnan yhteydessä. Kliinisesti ilmenee uneliaisuutta ja letargiaa, hypotensiota ja bradykardiaa, heikentynyttä hengitystä ja hypoventilaation merkkejä. Hoito- kohdennettu vaikutus etiologiseen tekijään ja magnesiumantagonistin - kalsiumsuolat - nimittäminen.

9.2.5. Kloorin vaihto

Kloori - solunulkoisen tilan pääanioni; on vastaavassa suhteessa natriumin kanssa. Se tulee kehoon natriumkloridin muodossa, joka hajottaa Na +:n ja C1:n mahassa." Kun se yhdistyy vedyn kanssa, kloori muodostaa suolahappoa.

Hypokloremia(plasman kloori alle 95 mmol/l) kehittyy pitkittyneen oksentelun, vatsakalvontulehduksen, pylorisen stenoosin, korkean suolitukoksen, lisääntyneen hikoilun yhteydessä. Hypokloremian kehittymiseen liittyy bikarbonaattipuskurin lisääntyminen ja alkaloosin ilmaantuminen. Kliinisesti ilmenee kuivumisena, hengitys- ja sydämen toimintahäiriönä. Kouristeleva tai koominen tila voi johtaa kuolemaan. Hoito koostuu kohdistetusta vaikutuksesta patogeneettiseen tekijään ja infuusiohoidon suorittamisesta klorideilla (pääasiassa natriumkloridivalmisteilla) laboratoriovalvonnassa.

Hyperkloremia(plasman klooria enemmän kuin PO mmol/l) kehittyy yleisen kuivumisen, nesteen heikentyneen poistumisen yhteydessä interstitiaalisesta tilasta (esim. akuutti munuaisten vajaatoiminta), lisääntyneeseen nesteen siirtymiseen verisuonikerroksesta interstitiumiin (hypoproteinemialla) ja suurien määrien klooria sisältävien nesteiden lisääminen. Hyperkloremian kehittymiseen liittyy veren puskurikapasiteetin heikkeneminen ja metabolisen asidoosin ilmaantuminen. Kliinisesti tämä ilmenee turvotuksen kehittymisenä. Perusperiaate hoitoon- vaikutus patogeneettiseen tekijään yhdistettynä syndroomahoitoon.

9.3. Vesi-elektrolyyttiaineenvaihdunnan häiriöiden päätyypit

Isotoninen nestehukka(plasmanatrium normaalirajoissa: 135-145 mmol/l) johtuu nestehukasta interstitiaalisessa tilassa. Koska interstitiaalisen nesteen elektrolyyttikoostumus on lähellä veriplasmaa, tapahtuu tasaista nesteen ja natriumin menetystä. Useimmiten isotoninen nestehukka kehittyy pitkittyneen oksentelun ja ripulin, akuuttien ja kroonisten maha-suolikanavan sairauksien, suolitukoksen, vatsakalvontulehduksen, haimatulehduksen, laajojen palovammojen, polyurian, diureettien hallitsemattoman käytön ja polytrauman yhteydessä. Kuivumiseen liittyy elektrolyyttien menetys ilman merkittävää muutosta plasman osmolaarisessa tilassa, joten merkittävää veden uudelleenjakautumista sektoreiden välillä ei tapahdu, mutta muodostuu hypovolemia. Kliinisesti

häiriöitä keskushemodynamiikassa havaitaan. Ihon turgor on vähentynyt, kieli on kuiva, oliguria jopa anuriaan. Hoito patogeneettinen; korvaushoito isotonisella natriumkloridiliuoksella (35-70 ml/kg/vrk). Infuusiohoito tulee suorittaa keskuslaskimopaineen ja tunnin välein tapahtuvan diureesin hallinnassa. Jos hypotonisen dehydraation korjaus suoritetaan metabolisen asidoosin taustalla, natriumia annetaan bikarbonaatin muodossa; metabolisessa alkaloosissa - kloridin muodossa.

Hypotoninen nestehukka(plasmanatrium alle 130 mmol/l) kehittyy tapauksissa, joissa natriumhävikki ylittää vesihäviön. Esiintyy suuria määriä elektrolyyttejä sisältävien nesteiden massiivisella häviöllä - toistuva oksentelu, runsas ripuli, runsas hikoilu, polyuria. Plasman natriumpitoisuuden laskuun liittyy sen osmolaarisuuden lasku, minkä seurauksena plasman vesi alkaa jakautua uudelleen soluihin, mikä aiheuttaa niiden turvotusta (sellunsisäistä hyperhydraatiota) ja luo vesivajetta interstitiaaliseen tilaan.

Kliinisesti tämä tila ilmenee ihon ja silmämunien alentuneena turgorina, heikentyneenä hemodynamiikkana ja tilavuutena, atsotemiana, heikentyneenä munuaisten ja aivojen toimintana sekä hemokonsentraationa. Hoito koostuu kohdistetusta vaikutuksesta patogeneettiseen tekijään ja aktiivisesta rehydraatiosta natriumia, kaliumia, magnesiumia (asasuola) sisältävillä liuoksilla. Hyperkalemiaan on määrätty disolia.

Hypertensiivinen nestehukka(plasmanatrium yli 150 mmol/l) johtuu ylimääräisestä vesihäviöstä natriumin hävikkiin verrattuna. Esiintyy akuutin munuaisten vajaatoiminnan polyuriavaiheessa, pitkittyneessä pakkodiureesissa ilman veden puutteen oikea-aikaista täydentämistä, kuumetta ja riittämätöntä vedenantoa parenteraalisen ravinnon aikana. Ylimääräinen veden menetys natriumiin nähden lisää plasman osmolaarisuutta, minkä seurauksena solunsisäistä nestettä alkaa kulkeutua verisuonikerrokseen. Muodostuu solunsisäinen dehydraatio (soludehydraatio, eksikoosi).

Kliiniset oireet- jano, heikkous, apatia, uneliaisuus ja vaikeissa tapauksissa - psykoosi, hallusinaatiot, kielen kuivuminen, kohonnut ruumiinlämpö, ​​oliguria, jossa on korkea suhteellinen virtsan tiheys, atsotemia. Aivosolujen kuivuminen aiheuttaa epäspesifisten neurologisten oireiden ilmaantumista: psykomotorista levottomuutta, sekavuutta, kouristuksia, kooman kehittymistä.

Hoito koostuu kohdistetusta vaikutuksesta patogeneettiseen tekijään ja solunsisäisen kuivumisen poistamiseen määräämällä glukoosiliuosinfuusioita insuliinin ja kaliumin kanssa. Suolojen, glukoosin, albumiinin ja diureettien hypertonisten liuosten antaminen on vasta-aiheista. Plasman natriumpitoisuuden ja osmolaarisuuden seuranta on välttämätöntä.

Isotoninen hyperhydraatio(plasmanatrium normaalialueella 135-145 mmol/l) esiintyy useimmiten sellaisten sairauksien taustalla, joihin liittyy turvotusoireyhtymä (krooninen sydämen vajaatoiminta, raskauden toksikoosi), isotonisten suolaliuosten liiallisen antamisen seurauksena. Tämän oireyhtymän esiintyminen on mahdollista myös maksakirroosin ja munuaissairauksien (nefroosi, glomerulonefriitti) taustalla. Isootonisen ylihydraation kehittymisen päämekanismi on ylimääräinen vesi ja suolat, joilla on normaali plasmaosmolaarisuus. Nesteretentiota tapahtuu pääasiassa interstitiaalisessa tilassa.

Kliinisesti tämä hyperhydraation muoto ilmenee valtimoverenpaineen ilmaantumisena, kehon painon nopeana nousuna, turvotusoireyhtymän, anasarkan kehittymisenä ja veren pitoisuusparametrien laskuna. Ylihydraation taustalla on vapaan nesteen puute.

Hoito koostuu diureettien käytöstä, joiden tarkoituksena on vähentää interstitiaalisen tilan määrää. Lisäksi 10 % albumiinia annetaan suonensisäisesti lisäämään plasman onkoottista painetta, minkä seurauksena interstitiaalinen neste alkaa kulkeutua verisuonikerrokseen. Jos tämä hoito ei anna toivottua vaikutusta, he turvautuvat hemodialyysiin veren ultrasuodatuksella.

Hyperhydraatio hypotoninen(plasmanatrium alle 130 mmol/l) tai "vesimyrkytys" voi ilmaantua erittäin suurten vesimäärien samanaikaisen nauttimisen yhteydessä, suolattomien liuosten pitkäaikaisessa laskimonsisäisessä annossa, kroonisesta sydämen vajaatoiminnasta johtuvaa turvotusta, maksakirroosia. maksa, ylijännitesuoja, ADH:n liikatuotanto. Päämekanismi on plasman osmolaarisuuden väheneminen ja nesteen siirtyminen soluihin.

Kliininen kuva ilmenee oksenteluna, usein löysänä, vetisenä ulosteena ja polyuriana. Lisätään merkkejä keskushermoston vaurioista: heikkous, heikkous, nopea väsymys, unihäiriöt, delirium, tajunnan heikkeneminen, kouristukset, kooma.

Hoito koostuu ylimääräisen veden poistamisesta kehosta mahdollisimman nopeasti: diureetteja määrätään samanaikaisesti suonensisäinen anto natriumkloridi, vitamiinit. Kalorinen ruokavalio vaaditaan. Tarvittaessa suoritetaan hemodialyysi veren ultrasuodatuksella.

ja Hyperhydraatio hypertensiivinen(plasmanatriumia enemmän 150 mmol/l) esiintyy, kun elimistöön tuodaan suuria määriä hypertonisia liuoksia munuaisten erittymistoiminnan säilymisen taustalla tai isotonisia liuoksia - potilaille, joiden munuaisten eritystoiminta on heikentynyt. Tilaan liittyy nesteen osmolaarisuuden lisääntyminen interstitiaalisessa tilassa, jota seuraa solusektorin kuivuminen ja lisääntynyt kaliumin vapautuminen siitä.

Kliininen kuva jolle on ominaista jano, ihon punoitus, kohonnut ruumiinlämpö, ​​verenpaine ja keskuslaskimopaine. Prosessin edetessä ilmenee merkkejä keskushermostovauriosta: mielenterveyshäiriöt, kouristukset, kooma.

Hoito- infuusiohoito inkluusiolla 5 % glukoosin ja albumiinin liuos diureesin stimuloinnin taustalla osmodiureeteilla ja salureeteilla. Käyttöaiheiden mukaan - hemodialyysi.

9.4 Happo-emästila

Happo-emästila(COS) on yksi tärkeimmistä kehon nesteiden biokemiallisen pysyvyyden komponenteista normaalien aineenvaihduntaprosessien perustana, jonka aktiivisuus riippuu elektrolyytin kemiallisesta reaktiosta.

CBS:lle on tunnusomaista vetyionien pitoisuus ja se on merkitty pH-symbolilla. Happamien liuosten pH on 1,0 - 7,0, emäksisten liuosten pH on 7,0 - 14,0. Asidoosi- pH:n siirtyminen happamalle puolelle tapahtuu happojen kertymisen tai emästen puutteen vuoksi. Alkaloosi- pH:n siirtyminen alkaliselle puolelle johtuu emästen ylimäärästä tai happopitoisuuden laskusta. pH:n pysyvyys on ihmiselämän välttämätön edellytys. pH on lopullinen, yleinen heijastus vetyionien (H +) pitoisuuden ja kehon puskurijärjestelmien tasapainosta. CBS:n tasapainon ylläpitäminen

suorittaa kaksi järjestelmää, jotka estävät veren pH:n muutoksen. Näitä ovat puskuri (fysikaalis-kemiallinen) ja fysiologiset järjestelmät CBS:n säätelyä varten.

9.4.1. Fysikaalis-kemialliset puskurijärjestelmät

Kehon fysikaalis-kemiallisia puskurijärjestelmiä tunnetaan neljä - bikarbonaatti, fosfaatti, veren proteiinipuskurijärjestelmä, hemoglobiini.

Bikarbonaattijärjestelmä joka muodostaa 10 % veren kokonaispuskurikapasiteetista, se on bikarbonaattien (HC0 3) ja hiilidioksidin (H 2 CO 3) suhde. Normaalisti se on 20:1. Bikarbonaattien ja hapon välisen vuorovaikutuksen lopputuote on hiilidioksidi (CO 2), joka hengitetään ulos. Bikarbonaattijärjestelmä on nopeimmin toimiva ja toimii sekä plasmassa että solunulkoisessa nesteessä.

Fosfaattijärjestelmä vie vähän tilaa puskurisäiliöissä (1 %), toimii hitaammin ja lopputuote - kaliumsulfaatti - erittyy munuaisten kautta.

Plasman proteiinit pH-tasosta riippuen ne voivat toimia sekä happoina että emäksinä.

Hemoglobiinipuskurijärjestelmä Sillä on tärkeä rooli happo-emästilan ylläpitämisessä (noin 70 % puskurikapasiteetista). Punasoluissa oleva hemoglobiini sitoo 20 % sisään tulevasta verestä, hiilidioksidista (C0 2) sekä hiilidioksidin (H 2 C0 3) dissosioitumisen seurauksena muodostuneista vetyioneista.

Bikarbonaattipuskuri on pääasiassa läsnä veressä ja kaikissa solunulkoisen nesteen osissa; plasmassa - bikarbonaatti-, fosfaatti- ja proteiinipuskurit; erytrosyyteissä - hiilikarbonaatti, proteiini, fosfaatti, hemoglobiini; virtsassa - fosfaatti.

9.4.2. Fysiologiset puskurijärjestelmät

Keuhkot säätelevät hiilihapon hajoamistuotteen CO 2 -pitoisuutta. CO 2:n kerääntyminen johtaa hyperventilaatioon ja hengenahdistukseen, jolloin ylimääräinen hiilidioksidi poistuu. Jos emäksiä on liikaa, tapahtuu päinvastainen prosessi - keuhkojen ventilaatio vähenee ja esiintyy bradypneaa. CO2:n ohella veren pH ja happipitoisuus ovat voimakkaita hengityskeskuksen ärsyttäjiä. pH-muutokset ja happipitoisuuden muutokset johtavat lisääntyneeseen keuhkojen ventilaatioon. Kaliumsuolat toimivat samalla tavalla, mutta kun K + -pitoisuus veriplasmassa kasvaa nopeasti, kemoreseptorien aktiivisuus vähenee ja keuhkojen ventilaatio vähenee. CBS:n hengityssäätely on nopea reagointijärjestelmä.

Munuaiset tukee CBS:ää useilla tavoilla. Hiilihappoanhydraasientsyymin vaikutuksesta, jota on suuria määriä munuaiskudokseen, CO 2 ja H 2 0 yhdistyvät muodostaen hiilihappoa. Hiilihappo hajoaa bikarbonaatiksi (HC0 3 ~) ja H+:ksi, joka yhdistyy fosfaattipuskurin kanssa ja erittyy virtsaan. Bikarbonaatit imeytyvät takaisin tubuluksiin. Kuitenkin, kun emäksiä on liikaa, reabsorptio vähenee, mikä johtaa lisääntyneeseen emästen erittymiseen virtsaan ja alkaloosin vähenemiseen. Jokainen titrattavien happojen tai ammoniumionien muodossa erittynyt millimooli H + lisää 1 mmol veriplasmaan

HC0 3 . Näin ollen H+:n erittyminen liittyy läheisesti HC03:n synteesiin. CBS:n munuaissäätely on hidasta ja vaatii useita tunteja tai jopa päiviä täyden korvauksen saamiseksi.

Maksa säätelee CBS:ää metaboloimalla alihapettuneita aineenvaihduntatuotteita, jotka tulevat maha-suolikanavasta, muodostaen ureaa typpipitoisista jätteistä ja poistamalla happoradikaaleja sapen mukana.

Ruoansulatuskanava Sillä on tärkeä paikka CBS:n pysyvyyden ylläpitämisessä nesteiden, ruoan ja elektrolyyttien otto- ja imeytymisprosessien korkean intensiteetin vuoksi. Minkä tahansa ruoansulatuksen osan rikkominen aiheuttaa CBS:n häiriöitä.

Kemialliset ja fysiologiset puskurijärjestelmät ovat tehokkaita ja tehokkaita mekanismeja CBS:n kompensoimiseksi. Tässä suhteessa pienimmätkin muutokset CBS:ssä osoittavat vakavia aineenvaihduntahäiriöitä ja sanelevat oikea-aikaisen ja kohdistetun korjaavan hoidon tarpeen. Yleisiä ohjeita CBS:n normalisoimiseksi ovat etiologisen tekijän poistaminen (hengitys- ja sydän- ja verisuonijärjestelmien patologia, vatsaelimet jne.), hemodynamiikan normalisointi - hypovolemian korjaaminen, mikroverenkierron palauttaminen, veren reologisten ominaisuuksien parantaminen, hengitysvajeen hoito, vatsan siirtoon asti. potilaalle koneellinen ventilaatio, vesi-elektrolyytti- ja proteiiniaineenvaihdunnan korjaus.

Jätevedenpuhdistamon indikaattorit määritetään Astrupa-tasapainotusmikromenetelmällä (interpolaatiolaskelmalla рС0 2) tai menetelmillä, joissa С0 2 on suora hapetus. Nykyaikaiset mikroanalysaattorit määrittävät automaattisesti kaikki CBS-arvot ja verikaasujen osittaiset jännitteet. Jätevedenpuhdistamon pääindikaattorit on esitetty taulukossa. 9.1.

Taulukko 9.1.CBS-ilmaisimet ovat normaaleja

Indeksi

Ominaista

Indikaattoriarvot

PaС0 2, mm Hg. Taide. Pa0 2, mm Hg. Taide.

AB, m mol/l SB, mmol/l

BB, mmol/l BE, mmol/l

Kuvaa liuoksen aktiivista reaktiota. Vaihtelee riippuen kehon puskurijärjestelmien kapasiteetista. Valtimoveren osittaisen jännityksen indikaattori C0 2 Valtimoveren osittaisen jännityksen indikaattori 0 2. Heijastaa hengityselinten toiminnallista tilaa Todellinen bikarbonaatti - bikarbonaatti-ionien pitoisuuden indikaattori Vakiobikarbonaatti - bikarbonaatti-ionien pitoisuuden indikaattori normaaleissa määritysolosuhteissa Plasmapuskuriemäkset, bikarbonaatin puskurikomponenttien kokonaisindikaattori, fosfaatti , proteiini- ja hemoglobiinijärjestelmät

Puskuriemästen yli- tai puutososoitus. Positiivinen arvo on emästen ylimäärä tai happojen puute. Negatiivinen arvo - emästen puute tai ylimäärä happoja

CBS-rikkomuksen tyypin arvioimiseksi normaalissa käytännön työssä käytetään indikaattoreita pH, PC0 2, P0 2, BE.

9.4.3. Happo-emäs-epätasapainon tyypit

CBS-häiriöitä on 4 päätyyppiä: metabolinen asidoosi ja alkaloosi; hengitysteiden asidoosi ja alkaloosi; Myös näiden yhdistelmät ovat mahdollisia.

A Aineenvaidunnallinen liiallinen happamuus- emäksen puute, mikä johtaa pH:n laskuun. Syyt: akuutti munuaisten vajaatoiminta, kompensoimaton diabetes (ketoasidoosi), sokki, sydämen vajaatoiminta (maitohappoasidoosi), myrkytys (salisylaatit, etyleeniglykoli, metyylialkoholi), ohutsuolen (pohjukaissuolen, haiman) fistelit, ripuli, lisämunuaisten vajaatoiminta. CBS-indikaattorit: pH 7,4-7,29, PaC02 40-28 Hg. Art., BE 0-9 mmol/l.

Kliiniset oireet- pahoinvointi, oksentelu, heikkous, tajunnan häiriöt, takypnea. Kliinisesti kohtalainen asidoosi (BE jopa -10 mmol/l) voi olla oireeton. Kun pH laskee arvoon 7,2 (alikompensaatiotila, sitten dekompensaatio), hengenahdistus lisääntyy. Kun pH laskee edelleen, hengitys- ja sydämen vajaatoiminta lisääntyy, ja hypoksinen enkefalopatia kehittyy koomaan asti.

Metabolisen asidoosin hoito:

Bikarbonaattipuskurijärjestelmän vahvistaminen - 4,2 % natriumbikarbonaattiliuoksen käyttöönotto (vasta-aiheet- hypokalemia, metabolinen alkaloosi, hypernatremia) suonensisäisesti perifeerisen tai keskuslaskimon kautta: laimentamaton, laimennettu 5-prosenttisella glukoosiliuoksella suhteessa 1:1. Liuoksen infuusionopeus on 200 ml 30 minuutissa. Tarvittava määrä natriumbikarbonaattia voidaan laskea kaavalla:

Natriumbikarbonaatin määrä mmol = BE ruumiinpaino, kg 0,3.

Ilman laboratoriotarkastusta, käytä enintään 200 ml/vrk, tipoittain, hitaasti. Liuosta ei saa antaa samanaikaisesti kalsiumia, magnesiumia sisältävien liuosten kanssa, eikä sitä saa sekoittaa fosfaattia sisältävien liuosten kanssa. Laktasolin siirto vaikutusmekanismin mukaan on samanlainen kuin natriumbikarbonaatin käyttö.

A Metabolinen alkaloosi- veren H + -ionien puutostila yhdessä emästen ylimäärän kanssa. Metabolista alkaloosia on vaikea hoitaa, koska se johtuu sekä ulkoisista elektrolyyttihäviöistä että solujen ja solunulkoisten ionisuhteiden häiriöistä. Tällaiset häiriöt ovat tyypillisiä massiiviselle verenhuolle, tulenkestävälle sokille, sepsikselle, vakavalle vesi- ja elektrolyyttihäviölle suolitukoksen aikana, vatsakalvontulehdukselle, haimanekroosille ja pitkäaikaisille suolen fisteleille. Melko usein metabolinen alkaloosi, joka on tämän potilasryhmän elämän kanssa yhteensopimattomien aineenvaihduntahäiriöiden viimeinen vaihe, tulee suoraksi kuolinsyyksi.

Metabolisen alkaloosin korjaamisen periaatteet. Metabolinen alkaloosi on helpompi ehkäistä kuin hoitaa. Ennaltaehkäiseviä toimenpiteitä ovat riittävä kaliumin antaminen verensiirtohoidon aikana ja solujen kaliumvajeen korjaaminen, voleemisten ja hemodynaamisten häiriöiden oikea-aikainen ja täydellinen korjaaminen. Vakiintuneen metabolisen alkaloosin hoidossa se on ensiarvoisen tärkeää

tämän tilan pääasiallisen patologisen tekijän poistaminen. Kaikentyyppisten vaihtojen tarkoituksellinen normalisointi suoritetaan. Alkaloosin lievitys saavutetaan antamalla suonensisäisesti proteiinivalmisteita, glukoosiliuoksia yhdessä kaliumkloridin kanssa ja suuria määriä vitamiineja. Isotonista natriumkloridiliuosta käytetään vähentämään solunulkoisen nesteen osmolaarisuutta ja eliminoimaan solujen kuivumista.

Hengityselinten asidoosi jolle on ominaista H + -ionien pitoisuuden nousu veressä (pH< 7,38), рС0 2 (>40 mmHg Art.), BE (= 3,5+12 mmol/l).

Hengitysteiden asidoosin syyt voivat olla hypoventilaatio keuhkoemfyseeman obstruktiivisen muodon seurauksena, keuhkoastma, heikentynyt ventilaatio heikentyneellä potilailla, laaja atelektaasi, keuhkokuume, akuutti keuhkovauriooireyhtymä.

Hengitysteiden asidoosin pääasiallinen kompensaatio tapahtuu munuaisten toimesta H +:n ja SG:n pakotetun erittymisen kautta, mikä lisää HC0 3:n uudelleenabsorptiota.

SISÄÄN kliininen kuva hengitysteiden asidoosia hallitsevat kallonsisäisen kohonneen verenpaineen oireet, jotka syntyvät liiallisen CO 2:n aiheuttaman aivojen verisuonten laajentumisen seurauksena. Progressiivinen hengitysteiden asidoosi johtaa aivoturvotukseen, jonka vakavuus vastaa hyperkapnian astetta. Stupor kehittyy usein ja etenee koomaan. Ensimmäiset hyperkapnian ja lisääntyvän hypoksian merkit ovat potilaan ahdistuneisuus, motorinen kiihtyneisyys, valtimoverenpaine, takykardia, jonka jälkeen siirtyy hypotensioon ja takyarytmiaan.

Hengitysteiden asidoosin hoito koostuu ensisijaisesti keuhkorakkuloiden ventilaation parantamisesta, atelektaasin, keuhko- tai vesirintakiven eliminoimisesta, trakeobronkiaalisen puun puhdistamisesta ja potilaan siirtämisestä koneelliseen ventilaatioon. Hoito on suoritettava kiireellisesti, ennen kuin hypoksia kehittyy hypoventilaatiosta.

ja Hengitysalkaloosi jolle on tunnusomaista pCO 2 -tason lasku alle 38 mm Hg. Taide. ja pH:n nousu yli 7,45-7,50 keuhkojen lisääntyneen ventilaation seurauksena sekä tiheydellä että syvyydellä (alveolaarinen hyperventilaatio).

Hengitysalkaloosin johtava patogeneettinen elementti on aivoverenkierron väheneminen aivoverenkierron lisääntymisen seurauksena, mikä on seurausta veren hiilidioksidin puutteesta. Alkuvaiheessa potilaalla voi esiintyä raajojen ja suun ympärillä olevien ihon parestesiaa, lihaskouristuksia raajoissa, lievää tai vaikeaa uneliaisuutta, päänsärkyä ja joskus syvempiä tajunnan häiriöitä, jopa koomaa.

Ennaltaehkäisy ja hoito hengitysteiden alkaloosin tarkoituksena on ensisijaisesti normalisoida ulkoinen hengitys ja vaikutus patogeneettiseen tekijään, joka aiheutti hyperventilaatiota ja hypokapniaa. Indikaatioita potilaan siirtämiseksi koneelliseen ventilaatioon ovat masennus tai spontaanin hengityksen puuttuminen sekä hengenahdistus ja hyperventilaatio.

9.5 Infuusiohoito neste- ja elektrolyyttihäiriöiden sekä happo-emästilanteen hoitoon

Infuusiohoito on yksi tärkeimmistä menetelmistä elintärkeiden elinten ja järjestelmien toimintahäiriöiden hoidossa ja ehkäisyssä kirurgisilla potilailla. Infuusion tehokkuus

hoito riippuu sen ohjelman pätevyydestä, infuusioväliaineen ominaisuuksista, farmakologiset ominaisuudet ja lääkkeen farmakokinetiikka.

varten diagnostiikka voleemishäiriöt ja rakentaminen infuusiohoito-ohjelmat pre- ja leikkauksen jälkeinen ajanjakso Ihon turgor, limakalvojen kosteus, ääreisvaltimon pulssin täyttö, syke ja verenpaine ovat tärkeitä. Leikkauksen aikana arvioidaan useimmiten ääreispulssin täyttöä, tuntidiureesia ja verenpaineen dynamiikkaa.

Hypervolemian ilmenemismuodot ovat takykardia, hengenahdistus, keuhkojen kosteusvärinä, syanoosi, vaahtoava yskös. Voleemishäiriöiden astetta heijastavat laboratoriotiedot - hematokriitti, valtimoveren pH, virtsan suhteellinen tiheys ja osmolaarisuus, natriumin ja kloorin pitoisuus virtsassa, natrium plasmassa.

Laboratoriomerkkeihin kuivuminen Näitä ovat hematokriitin nousu, etenevä metabolinen asidoosi, virtsan suhteellinen tiheys yli 1010, virtsan Na + -pitoisuuden lasku alle 20 mEq/l ja virtsan hyperosmolaarisuus. Hypervolemialle tyypillisiä laboratoriomerkkejä ei ole. Hypervolemia voidaan diagnosoida rintakehän röntgentietojen perusteella - lisääntynyt vaskulaarinen keuhkokuvio, interstitiaalinen ja alveolaarinen keuhkopöhö. CVP arvioidaan kulloisenkin kliinisen tilanteen mukaan. Paljastavin on tilavuuskuormitustesti. CVP:n lievä nousu (1-2 mm Hg) nopean kristalloidiliuoksen (250-300 ml) infuusion jälkeen viittaa hypovolemiaan ja tarpeeseen lisätä infuusiohoitoa. Ja päinvastoin, jos testin jälkeen keskuslaskimopaineen nousu ylittää 5 mm Hg. Art., on tarpeen vähentää infuusiohoidon nopeutta ja rajoittaa sen määrää. Infuusiohoitoon kuuluu kolloidi- ja kristalloidiliuosten suonensisäinen antaminen.

A Kristalloidiliuokset - pienen molekyylipainon ionien (suolat) vesiliuokset tunkeutuvat nopeasti verisuonen seinämään ja jakautuvat solunulkoiseen tilaan. Ratkaisun valinta riippuu korvattavan nestehäviön luonteesta. Veden hävikki korvataan hypotonisilla ratkaisuilla, joita kutsutaan ylläpitoratkaisuiksi. Veden ja elektrolyyttien puute korvataan isotonisilla elektrolyyttiliuoksilla, joita kutsutaan korvausliuoksiksi.

Kolloidiset liuokset Ne perustuvat gelatiiniin, dekstraaniin, hydroksietyylitärkkelys- ja polyetyleeniglykoliin, ne ylläpitävät plasman kolloid-osmoottista painetta ja kiertävät verisuonikerroksessa tarjoten voleemisen, hemodynaamisen ja reologisen vaikutuksen.

Perioperatiivisella jaksolla infuusiohoidon avulla tyydytetään fysiologiset nestetarpeet (ylläpitohoito), samanaikainen nestevaje ja leikkaushaavan kautta menetykset. Infuusioliuoksen valinta riippuu menetettävän nesteen koostumuksesta ja luonteesta - hiki, maha-suolikanavan sisältö. Leikkauksensisäinen veden ja elektrolyyttien menetys johtuu haihtumisesta leikkaushaavan pinnalta laajojen kirurgisten toimenpiteiden aikana ja riippuu haavan pinnan pinta-alasta ja leikkauksen kestosta. Siten leikkauksensisäiseen nestehoitoon kuuluu fysiologisten perusnesteen tarpeiden täyttäminen, leikkausta edeltävien puutteiden ja operatiivisten menetysten poistaminen.

Taulukko 9.2. Elektrolyyttipitoisuus ruoansulatuskanavassa

Päivittäin

tilavuus, ml

Mahalaukun mehu

Haimamehu

Suoliston mehu

Vuoto ileostoman kautta

Vuoto ripulista

Vuoto kolostomiasta

Veden tarve määritetään tuloksena olevan nesteen puutteen tarkan arvioinnin perusteella, ottaen huomioon munuaisten ja munuaisten ulkopuoliset menetykset.

Tätä tarkoitusta varten päivittäisen diureesin määrä lasketaan yhteen: V, - oikea arvo on 1 ml/kg/h; V 2 - menetykset oksentamisen, ulosteen ja ruoansulatuskanavan sisällön kautta; V 3 - tyhjennyspoisto; P - hikoilun menetys ihon ja keuhkojen läpi (10-15 ml/kg/vrk), ottaen huomioon jatkuvat T-häviöt kuumeen aikana (ruumiinlämpötilan noustessa 1 °C yli 37 °C, menetykset ovat 500 ml päivässä). Näin ollen päivittäinen kokonaisvesivaje lasketaan kaavalla:

E = V, + V2 + V3 + P + T (ml).

Hypo- tai ylihydraation estämiseksi on tarpeen kontrolloida kehon nesteen määrää, erityisesti sen, joka sijaitsee solunulkoisessa tilassa:

OVZh = ruumiinpaino, kg 0,2, muuntokerroin Hematocrit - Hematocrit

Puutos = todellinen oikea ruumiinpaino, kg Hematokriitti oikea 5

Olennaisen elektrolyyttivajeen laskeminen(K + , Na +) tuotetaan ottaen huomioon niiden virtsassa, maha-suolikanavan (GIT) ja vedenpoistoväliaineen sisältö; pitoisuusindikaattoreiden määrittäminen - yleisesti hyväksyttyjen biokemiallisten menetelmien mukaisesti. Mikäli kaliumia, natriumia, klooria ei voida määrittää mahan sisällöstä, häviöt voidaan arvioida ensisijaisesti ottaen huomioon indikaattoreiden pitoisuuksien vaihtelut seuraavissa rajoissa: Na + 75-90 mmol/l; K + 15-25 mmol/l, SG 130 mmol/l asti, kokonaistyppi 3-5,5 g/l.

Eli elektrolyyttien kokonaishäviö päivässä on:

E = V, C, + V 2 C 2 + V 3 C 3 g,

jossa V] on päivittäinen diureesi; V 2 - maha-suolikanavan eritteen määrä oksentelun aikana, ulosteen kanssa, putken kautta sekä fistelihäviöt; V 3 - poisto tyhjennyksen kautta vatsaontelosta; C, C 2, C 3 - pitoisuusindikaattorit näissä ympäristöissä, vastaavasti. Laskettaessa voit viitata taulukon tietoihin. 9.2.

Muunnettaessa häviöarvoa mmol/l:sta (SI-järjestelmä) grammoiksi, on suoritettava seuraavat muunnokset:

K+, g = mmol/l 0,0391.

Na+, g = mmol/l 0,0223.

9.5.1. Kristalloidiliuosten ominaisuudet

Vesi-elektrolyytti- ja happo-emäs-homeostaasia sääteleviin aineisiin kuuluvat elektrolyyttiliuokset ja osmodiureetit. Elektrolyyttiliuokset käytetään korjaamaan vesi-, elektrolyytti-, vesi-elektrolyytti-, happo-emäs- (metabolinen asidoosi), vesi-elektrolyyttiaineenvaihdunnan ja happo-emästilan häiriöitä (metabolinen asidoosi). Elektrolyyttiliuosten koostumus määrittää niiden ominaisuudet - osmolaarisuuden, isotonisuuden, ionisuuden, varaemäksisuuden. Mitä tulee elektrolyyttiliuosten veren osmolaarisuuteen, niillä on iso-, hypo- tai hyperosmolaarinen vaikutus.

    Isoosmolaarinen vaikutus - Isomolaarisella liuoksella (Ringerin liuos, Ringerin asetaatti) annettu vesi jakautuu suonensisäisten ja suonen ulkopuolisten tilojen välillä 25 %:75 % (voleeminen vaikutus on 25 % ja kestää noin 30 minuuttia). Nämä liuokset on tarkoitettu isotoniseen dehydraatioon.

    Hypoosmolaarinen vaikutus - yli 75 % elektrolyyttiliuoksen (disoli, acesoli, 5 % glukoosiliuos) mukana tulevasta vedestä menee ekstravaskulaariseen tilaan. Nämä liuokset on tarkoitettu hypertensiiviseen dehydraatioon.

    Hyperosmolaarinen vaikutus - vesi ekstravaskulaarisesta tilasta tulee verisuonipohjaan, kunnes liuoksen hyperosmolaarisuus on saatettu veren osmolaarisuuteen. Nämä liuokset on tarkoitettu hypotoniseen dehydraatioon (10 % natriumkloridiliuos) ja hyperhydraatioon (10 % ja 20 % mannitolia).

Liuoksen elektrolyyttipitoisuudesta riippuen ne voivat olla isotonisia (0,9 % natriumkloridiliuos, 5 % glukoosiliuos), hypotonisia (disoli, acesoli) ja hypertonisia (4 % kaliumkloridiliuos, 10 % natriumkloridi, 4,2 % ja 8,4 % % natriumbikarbonaattiliuos). Jälkimmäisiä kutsutaan elektrolyyttikonsentraateiksi, ja niitä käytetään lisäaineena infuusioliuoksiin (5 % glukoosiliuos, Ringerin asetaattiliuos) juuri ennen antoa.

Liuoksen ionien lukumäärästä riippuen ne erotetaan monoionisiin (natriumkloridiliuos) ja polyionisiin (Ringerin liuos jne.).

Varaemäksisten kantaja-aineiden (hiilikarbonaatti, asetaatti, laktaatti ja fumaraatti) lisääminen elektrolyyttiliuoksiin mahdollistaa metabolisen asidoosin häiriöiden korjaamisen.

Natriumkloridiliuos 0,9 % annetaan suonensisäisesti perifeerisen tai keskuslaskimon kautta. Antonopeus on 180 tippaa/min eli noin 550 ml/70 kg/h. Keskimääräinen annos aikuiselle potilaalle on 1000 ml/vrk.

Käyttöaiheet: hypotoninen nestehukka; Na+:n ja O:n tarpeen tyydyttäminen; hypokloreeminen metabolinen alkaloosi; hyperkalsemia.

Vasta-aiheet: hypertensiivinen kuivuminen; hypernatremia; hyperkloremia; hypokalemia; hypoglykemia; hyperkloreeminen metabolinen asidoosi.

Mahdolliset komplikaatiot:

    hypernatremia;

    hyperkloremia (hyperkloreeminen metabolinen asidoosi);

    liiallinen nesteytys (keuhkopöhö).

g Ringerin asetaattiliuos- isotoninen ja isoioninen liuos, joka annetaan suonensisäisesti. Antonopeus on 70-80 tippaa/min tai 30 ml/kg/h;

tarvittaessa jopa 35 ml/min. Keskimääräinen annos aikuiselle potilaalle on 500-1000 ml/vrk; tarvittaessa jopa 3000 ml/vrk.

Käyttöaiheet: veden ja elektrolyyttien menetys maha-suolikanavasta (oksentelu, ripuli, fistelit, suolitukos, peritoniitti, haimatulehdus jne.); virtsan kanssa (polyuria, isostenuria, pakkodiureesi);

Isotoninen dehydraatio metabolisella asidoosilla - asidoosin viivästynyt korjaus (verenhukkaa, palovammat).

Vasta-aiheet:

    hypertensiivinen ylihydraatio;

  • hypernatremia;

    hyperkloremia;

    hyperkalsemia.

Komplikaatiot:

    liiallinen nesteytys;

  • hypernatremia;

    hyperkloremia.

A Yonosteril- isotoninen ja isoioninen elektrolyyttiliuos annetaan laskimoon perifeerisen tai keskuslaskimon kautta. Antonopeus on 3 ml/kg ruumiinpainoa tai 60 tippaa/min tai 210 ml/70 kg/h; tarvittaessa jopa 500 ml/15 min. Keskimääräinen annos aikuiselle on 500-1000 ml/vrk. Vakavissa tai kiireellisissä tapauksissa jopa 500 ml 15 minuutissa.

Käyttöaiheet:

eri alkuperää oleva solunulkoinen (isotoninen) nestehukka (oksentelu, ripuli, fistelit, vedenpoistot, suolen tukkeuma, peritoniitti, haimatulehdus jne.); polyuria, isostenuria, pakkodiureesi;

Ensisijainen plasmakorvaus plasmahäviön ja palovammojen varalta. Vasta-aiheet: hypertensiivinen ylihydraatio; turvotus; raskas

munuaisten vajaatoiminta.

Komplikaatiot: ylihydraatio.

Laktosoli- isotoninen ja isoioninen elektrolyyttiliuos annetaan laskimoon perifeerisen tai keskuslaskimon kautta. Antonopeus on 70-80 tippaa/min eli noin 210 ml/70 kg/h; tarvittaessa jopa 500 ml/15 min. Keskimääräinen annos aikuiselle on 500-1000 ml/vrk; tarvittaessa jopa 3000 ml/vrk.

Käyttöaiheet:

    veden ja elektrolyyttien menetys maha-suolikanavasta (oksentelu, ripuli, fistelit, vedenpoistot, suolitukokset, vatsakalvontulehdus, haimatulehdus jne.); virtsan kanssa (polyuria, isostenuria, pakkodiureesi);

    isotoninen nestehukka metabolisella asidoosilla (asidoosin nopea ja viivästynyt korjaus) - verenhukka, palovammat.

Vasta-aiheet: hypertensiivinen ylihydraatio; alkaloosi; hypernatremia; hyperkloremia; hyperkalsemia; hyperlaktatemia.

Komplikaatiot: liiallinen nesteytys; alkaloosi; hypernatremia; hyperkloremia; hyperlaktatemia.

Acesol- hypoosmolaarinen liuos sisältää Na +, C1" ja asetaatti-ioneja. Annostetaan suonensisäisesti perifeerisen tai keskuslaskimon kautta (virtaus

tai tippua). Aikuisen vuorokausiannos on yhtä suuri kuin päivittäinen veden ja elektrolyyttien tarve plus "/2" vesivaje sekä jatkuvat patologiset menetykset.

Käyttöaiheet: hypertensiivinen dehydraatio yhdistettynä hyperkalemiaan ja metaboliseen asidoosiin (asidoosin viivästynyt korjaus).

Vasta-aiheet: hypotoninen nestehukka; hypokalemia; ylihydraatio.

Komplikaatio: hyperkalemia.

A Natriumbikarbonaattiliuos 4.2% metabolisen asidoosin nopeaan korjaamiseen. Anna suonensisäisesti laimentamattomana tai laimennettuna 5 % glukoosiliuosta suhteessa 1:1, annos riippuu ionogrammista ja CBS-tiedoista. Laboratoriokontrollin puuttuessa annostus on enintään 200 ml/vrk hitaasti, tipoittain. 4,2 % natriumbikarbonaattiliuosta ei saa antaa samanaikaisesti kalsiumia, magnesiumia sisältävien liuosten kanssa, eikä sitä saa sekoittaa fosfaattia sisältävien liuosten kanssa. Lääkkeen annos voidaan laskea kaavalla:

1 ml 4,2 % liuosta (0,5 moolia) = BE ruumiinpaino (kg) 0,6.

Indikaatiot - aineenvaidunnallinen liiallinen happamuus.

Vasta-aiheet- hypokalemia, metabolinen alkaloosi, hypernatremia.

Osmodiureetit(mannitoli). 75-100 ml 20-prosenttista mannitolia annetaan laskimoon 5 minuutin aikana. Jos virtsan määrä on alle 50 ml/h, seuraavat 50 ml annetaan suonensisäisesti.

9.5.2. Hypo- ja ylihydraation infuusiohoidon pääohjeet

1. Infuusiohoito kuivuminen tulee ottaa huomioon sen tyyppi (hypertoninen, isotoninen, hypotoninen) sekä:

    "kolmannen tilan" tilavuus; pakottaa diureesi; hypertermia; hyperventilaatio, avoimet haavat; hypovolemia.

2. Infuusiohoito ylihydraatio tulee ottaa huomioon sen tyyppi (hypertoninen, isotoninen, hypotoninen) sekä:

    fysiologinen päivittäinen veden ja elektrolyyttien tarve;

    aiempi veden ja elektrolyyttien puutos;

    jatkuva patologinen nesteen menetys eritteiden kanssa;

    "kolmannen tilan" tilavuus; pakottaa diureesi; hypertermia, hyperventilaatio; avoimet haavat; hypovolemia.